Sie sind auf Seite 1von 47

RULE 111: PROSECUTION OF CIVIL ACTION

A. Institution of Criminal and Civil Actions

 Revised Penal Code, Title 5, Civil Liability

Chapter One
PERSON CIVILLY LIABLE FOR FELONIES

Article 100. Civil liability of a person guilty of felony. - Every person criminally liable for a felony is also
civilly liable.

Article 101. Rules regarding civil liability in certain cases. - The exemption from criminal liability
established in subdivisions 1, 2, 3, 5 and 6 of Article 12 and in subdivision 4 of Article 11 of this Code
does not include exemption from civil liability, which shall be enforced subject to the following rules:

First. In cases of subdivisions 1, 2, and 3 of Article 12, the civil liability for acts committed by an imbecile
or insane person, and by a person under nine years of age, or by one over nine but under fifteen years of
age, who has acted without discernment, shall devolve upon those having such person under their legal
authority or control, unless it appears that there was no fault or negligence on their part.

Should there be no person having such insane, imbecile or minor under his authority, legal guardianship
or control, or if such person be insolvent, said insane, imbecile, or minor shall respond with their own
property, excepting property exempt from execution, in accordance with the civil law.

Second. In cases falling within subdivision 4 of Article 11, the persons for whose benefit the harm has
been prevented shall be civilly liable in proportion to the benefit which they may have received.

The courts shall determine, in sound discretion, the proportionate amount for which each one shall be
liable.

When the respective shares cannot be equitably determined, even approximately, or when the liability
also attaches to the Government, or to the majority of the inhabitants of the town, and, in all events,
whenever the damages have been caused with the consent of the authorities or their agents,
indemnification shall be made in the manner prescribed by special laws or regulations.

Third. In cases falling within subdivisions 5 and 6 of Article 12, the persons using violence or causing the
fears shall be primarily liable and secondarily, or, if there be no such persons, those doing the act shall be
liable, saving always to the latter that part of their property exempt from execution.

Article 102. Subsidiary civil liability of innkeepers, tavernkeepers and proprietors of establishments. - In
default of the persons criminally liable, innkeepers, tavernkeepers, and any other persons or corporations
shall be civilly liable for crimes committed in their establishments, in all cases where a violation of
municipal ordinances or some general or special police regulation shall have been committed by them or
their employees.

Innkeepers are also subsidiarily liable for the restitution of goods taken by robbery or theft within their
houses from guests lodging therein, or for the payment of the value thereof, provided that such guests
shall have notified in advance the innkeeper himself, or the person representing him, of the deposit of
such goods within the inn; and shall furthermore have followed the directions which such innkeeper or his
representative may have given them with respect to the care and vigilance over such goods. No liability
shall attach in case of robbery with violence against or intimidation of persons unless committed by the
innkeeper's employees.
Article 103. Subsidiary civil liability of other persons. - The subsidiary liability established in the next
preceding article shall also apply to employers, teachers, persons, and corporations engaged in any kind
of industry for felonies committed by their servants, pupils, workmen, apprentices, or employees in the
discharge of their duties.

Chapter Two
WHAT CIVIL LIABILITY INCLUDES

Article 104. What is included in civil liability. - The civil liability established in Articles 100, 101, 102, and
103 of this Code includes:

1. Restitution;

2. Reparation of the damage caused;

3. Indemnification for consequential damages.

Article 105. Restitution; How made. - The restitution of the thing itself must be made whenever possible,
with allowance for any deterioration, or diminution of value as determined by the court.

The thing itself shall be restored, even though it be found in the possession of a third person who has
acquired it by lawful means, saving to the latter his action against the proper person, who may be liable to
him.

This provision is not applicable in cases in which the thing has been acquired by the third person in the
manner and under the requirements which, by law, bar an action for its recovery.

Article 106. Reparation; How made. - The court shall determine the amount of damage, taking into
consideration the price of the thing, whenever possible, and its special sentimental value to the injured
party, and reparation shall be made accordingly.

Article 107. Indemnification; What is included. - Indemnification for consequential damages shall include
not only those caused the injured party, but also those suffered by his family or by a third person by
reason of the crime.

Article 108. Obligation to make restoration, reparation for damages, or indemnification for consequential
damages and actions to demand the same; Upon whom it devolves. - The obligation to make restoration
or reparation for damages and indemnification for consequential damages devolves upon the heirs of the
person liable.

The action to demand restoration, reparation, and indemnification likewise descends to the heirs of the
person injured.

Article 109. Share of each person civilly liable. - If there are two or more persons civilly liable for a felony,
the courts shall determine the amount for which each must respond.

Article 110. Several and subsidiary liability of principals, accomplices and accessories of a felony;
Preference in payment. - Notwithstanding the provisions of the next preceding article, the principals,
accomplices, and accessories, each within their respective class, shall be liable severally (in solidum)
among themselves for their quotas, and subsidiaries for those of the other persons liable.

The subsidiary liability shall be enforced, first against the property of the principals; next, against that of
the accomplices, and, lastly, against that of the accessories.
Whenever the liability in solidum or the subsidiary liability has been enforced, the person by whom
payment has been made shall have a right of action against the others for the amount of their respective
shares.

Article 111. Obligation to make restitution in certain cases. - Any person who has participated
gratuitously in the proceeds of a felony shall be bound to make restitution in an amount equivalent to the
extent of such participation.

Chapter Three
EXTINCTION AND SURVIVAL OF CIVIL LIABILITY

Article 112. Extinction of civil liability. - Civil liability established in Articles 100, 101, 102, and 103 of this
Code shall be extinguished in the same manner as obligations, in accordance with the provisions of the
Civil Law.

Article 113. Obligation to satisfy civil liability. - Except in case of extinction of his civil liability as provided
in the next preceding article the offender shall continue to be obliged to satisfy the civil liability resulting
from the crime committed by him, notwithstanding the fact that he has served his sentence consisting of
deprivation of liberty or other rights, or has not been required to serve the same by reason of amnesty,
pardon, commutation of sentence or any other reason.

 Civil Code of the Philippines

Article 29. When the accused in a criminal prosecution is acquitted on the ground that his guilt has not
been proved beyond reasonable doubt, a civil action for damages for the same act or omission may be
instituted. Such action requires only a preponderance of evidence. Upon motion of the defendant, the
court may require the plaintiff to file a bond to answer for damages in case the complaint should be found
to be malicious.

If in a criminal case the judgment of acquittal is based upon reasonable doubt, the court shall so declare.
In the absence of any declaration to that effect, it may be inferred from the text of the decision whether or
not the acquittal is due to that ground.

Article 32. Any public officer or employee, or any private individual, who directly or indirectly obstructs,
defeats, violates or in any manner impedes or impairs any of the following rights and liberties of another
person shall be liable to the latter for damages:

(1) Freedom of religion;

(2) Freedom of speech;

(3) Freedom to write for the press or to maintain a periodical publication;

(4) Freedom from arbitrary or illegal detention;

(5) Freedom of suffrage;

(6) The right against deprivation of property without due process of law;

(7) The right to a just compensation when private property is taken for public use;

(8) The right to the equal protection of the laws;


(9) The right to be secure in one's person, house, papers, and effects against unreasonable
searches and seizures;

(10) The liberty of abode and of changing the same;

(11) The privacy of communication and correspondence;

(12) The right to become a member of associations or societies for purposes not contrary to law;

(13) The right to take part in a peaceable assembly to petition the Government for redress of
grievances;

(14) The right to be a free from involuntary servitude in any form;

(15) The right of the accused against excessive bail;

(16) The right of the accused to be heard by himself and counsel, to be informed of the nature
and cause of the accusation against him, to have a speedy and public trial, to meet the witnesses
face to face, and to have compulsory process to secure the attendance of witness in his behalf;

(17) Freedom from being compelled to be a witness against one's self, or from being forced to
confess guilt, or from being induced by a promise of immunity or reward to make such
confession, except when the person confessing becomes a State witness;

(18) Freedom from excessive fines, or cruel and unusual punishment, unless the same is
imposed or inflicted in accordance with a statute which has not been judicially declared
unconstitutional; and

(19) Freedom of access to the courts.

In any of the cases referred to in this article, whether or not the defendant's act or omission
constitutes a criminal offense, the aggrieved party has a right to commence an entirely separate
and distinct civil action for damages, and for other relief. Such civil action shall proceed
independently of any criminal prosecution (if the latter be instituted), and may be proved by a
preponderance of evidence.

The indemnity shall include moral damages. Exemplary damages may also be adjudicated.

The responsibility herein set forth is not demandable from a judge unless his act or omission constitutes a
violation of the Penal Code or other penal statute.

Article 33. In cases of defamation, fraud, and physical injuries a civil action for damages, entirely
separate and distinct from the criminal action, may be brought by the injured party. Such civil action shall
proceed independently of the criminal prosecution, and shall require only a preponderance of evidence.

Article 34. When a member of a city or municipal police force refuses or fails to render aid or protection
to any person in case of danger to life or property, such peace officer shall be primarily liable for
damages, and the city or municipality shall be subsidiarily responsible therefor. The civil action herein
recognized shall be independent of any criminal proceedings, and a preponderance of evidence shall
suffice to support such action.
Article 2176. Whoever by act or omission causes damage to another, there being fault or negligence, is
obliged to pay for the damage done. Such fault or negligence, if there is no pre-existing contractual
relation between the parties, is called a quasi-delict and is governed by the provisions of this Chapter.

Cases:

1. Sec. Of Justice V. Hon. Lantion (2000)

FACTS:
In accordance to "Extradition Treaty Between the Government of the Republic of the Philippines and the
Government of the United States of America" (RP-US Extradition Treaty), the Department of Justice
received from the Department of Foreign Affairs U.S. Note Verbale No. 0522 containing a request for the
extradition of Mark Jimenez to the United States attached with the Grand Jury Indictment, the warrant of
arrest issued by the U.S. District Court, Southern District of Florida, and other supporting documents on
June 18, 1999.

The Department of Justice denied the request for extradition.

Mr. Jimenez filed with filed with the Regional Trial Court of the National Capital Judicial Region a petition
presided over by the Honorable Ralph C. Lantion against the Secretary of Justice, the Secretary of
Foreign Affairs, and the Director of the National Bureau of Investigation:

i. mandamus to compel the Department to furnish the extradition documents


ii. certiorari to set aside Department’s letter dated July 13, 1999 denying his request
iii. prohibition to restrain the Department from considering the extradition request and from filing an
extradition petition in court
iv. enjoin the Secretary of Foreign Affairs and the Director of the NBI from performing any act directed to
the extradition
v. application for the issuance of a temporary restraining order and a writ of preliminary injunction

Honorable Ralph C. Lantion ordered the Secretary of Justice, the Secretary of Foreign
Affairs and the Director of the National Bureau of Investigation to maintain the status quo by refraining
from committing the acts complained of, from conducting further proceedings in connection with the
request of the United States Government, from filing the corresponding Petition with a Regional Trial
court and from performing any act directed to the extradition for a period of 20 days from service of the
order.

Hon. Hilario G. Davide, Jr., Chief Justice of the Supreme Court of the Philippines ordered Hon. Lantion to
cease and desist from enforcing the order. Due to transcendental importance, the Court brushed aside
peripheral procedural matters which concern the proceedings in Civil Case No. 99-94684 and the TRO
and proceded on the issues.

ISSUE:
Whether or not the evaluation procedure is not a preliminary investigation nor akin to preliminary
investigation of criminal cases.

RULING:
No.
The Extradition Request (Sec. 4. PD 1069) is made by the Foreign Diplomat of the
Requesting State, addressed to the Secretary of Foreign Affairs. The Secretary of Foreign Affairs has the
executive authority to conduct the evaluation process which, just like the extradition proceedings proper,
belongs to a class by itself or is sui generis. It is not a criminal investigation but it is also erroneous to say
that it is purely an exercise of ministerial functions. At such stage, the executive authority has the power:
1) to make a technical assessment of the completeness and sufficiency of the extradition papers in form
and substance
2) to outrightly deny the request if on its face and on the face of the supporting documents the crimes
indicated are not extraditable
3) to make a determination whether or not the request is politically motivated, or that the offense is a
military one which is not punishable under non-military penal legislation.

The process may be characterized as an investigative or inquisitorial process (NOT an exercise of an


administrative body's quasi-judicial power) (Sec. 5. PD 1069 and Pars. 2 and 3, Art. 7 of the RP-US
Extradition Treaty) that is indispensable to prosecution. The power of investigation consists in gathering,
organizing and analyzing evidence, which is a useful aid or tool in an administrative agency's
performance of its rule-making or quasi-judicial functions.

The evaluation process partakes of the nature of a criminal investigation. Similar to the evaluation stage
of extradition proceedings, a preliminary investigation, which may result in the filing of an information
against the respondent, can possibly lead to his arrest, and to the deprivation of his liberty. The
characterization of a treaty in Wright was in reference to the applicability of the prohibition against an ex
post facto law. It had nothing to do with the denial of the right to notice, information, and hearing.

In this case, the extradition request was delivered to the Department of Foreign Affairs on June 17, 1999
(the following day the Department of Justice received the request). Thus, the Department of Foreign
Affairs failed to discharge its duty of evaluating the same and its accompanying documents.

2. People vs. Bayotas

FACTS:
Rogelio Bayotas, accused and charged with Rape, died on February 4, 1992 due to cardio respiratory
arrest. The Solicitor General then submitted a comment stating that the death of the accused does
not excuse him from his civil liability (supported by the Supreme Court’s decision in People vs
Sendaydiego). On the other hand, the counsel of the accused claimed that in the Supreme Court’s
decision in People vs Castillo, civil liability is extinguished if accused should die before the
final judgement is rendered.

ISSUE:
Whether or not the death of the accused pending appeal of his conviction extinguish his civil liability.

RULING:
The Court decided on this case through stating the cases of Castillo and Sendaydiego. In the Castillo
case, the Court said that civil liability is extinguished only when death of the accused occurred before the
final judgement. Judge Kapunan further stated that civil liability is extinguished because there will be “no
party defendant” in the case. There will be no civil liability if criminal liability does not exist. Further, the
Court stated “it is, thus, evident that… the rule established was that the survival of the
civil liability depends on whether the same can be predicated on the sources of obligations other than
delict.

In the Sendaydiego case, the Court issued Resolution of July 8, 1977 where it states that civil liability will
only survive if death came after the final judgement of the CFI of Pangasinan. However, Article 30 of the
Civil Code could not possibly lend support to the ruling in Sendaydiego. Civil liability ex delicto is
extinguished by the death of the accused while his conviction is on appeal. The Court also gave a
summary on which cases should civil liability be extinguished, to wit:
Death of the accused pending appeal of his conviction extinguishes his criminal liability as well as the
civil liability based solely thereon. Therefore, Bayotas’s death extinguished his criminal and
civil liability based solely on the act complained of.
B. When separate action is suspended

Cases:

1. Manatan vs. CA

FACTS:
In 1982, accused Manantan, being then the driver and person-in-charge of an automobile, willfully and
unlawfully drove and operated the same in a negligent, careless and imprudent manner, without due
regard to traffic laws without taking the necessary precaution to prevent accident to person and damage
to property, causing said automobile to sideswipe a passenger jeep resulting to the death of Ruben
Nicolas a passenger of said automobile. Manantan was acquitted by the trial court of homicide through
reckless imprudence without a ruling on his civil liability. On appeal from the civil aspect of the judgment,
the appellate court found petitioner Manantan civilly liable and ordered him to indemnify private
respondents Marcelino Nicolas and Maria Nicolas P104,400.00 finding accused intoxicated of alcohol at
the time of the accident.

ISSUE:
W/N the acquittal extinguished the civil liability.

RULING:
Decision affirmed.

While the trial court found that petitioner's guilt had not been proven beyond reasonable doubt, it did not
state in clear and unequivocal terms that petitioner was not recklessly imprudent or negligent. The trial
court acquitted accused on reasonable doubt. Since civil liability is not extinguished in criminal cases, if
the acquittal is based on reasonable doubt, the Court of Appeals had to review the findings of the trial
court to determine if there was a basis for awarding indemnity and damages.

This is the situation contemplated in Article 29 of the Civil Code where the civil action for damages is "for
the same act or omission." Although the two actions have different purposes, the matters discussed in the
civil case are similar to those discussed in the criminal case. However, the judgment in the criminal
proceeding cannot be read in evidence in the civil action to establish any fact there determined, even
though both actions involve the same act or omission.

The reason for this rule is that the parties are not the same and secondarily, different rules of evidence
are applicable. Hence, notwithstanding herein petitioner's acquittal, the Court of Appeals in determining
whether Article 29 applied, was not precluded from looking into the question of petitioner's negligence or
reckless imprudence.

C. When civil action may proceed independently

Section 3, Rule 111:


When civil action may proceeded independently. — In the cases provided for in Articles 32, 33, 34 and
2176 of the Civil Code of the Philippines, the independent civil action may be brought by the offended
party. It shall proceed independently of the criminal action and shall require only a preponderance of
evidence. In no case, however, may the offended party recover damages twice for the same act or
omission charged in the criminal action.

Art. 32,Civil Code of the Philippines. Any public officer or employee, or any private individual, who
directly or indirectly obstructs, defeats, violates or in any manner impedes or impairs any of the following
rights and liberties of another person shall be liable to the latter for damages:

(1) Freedom of religion;


(2) Freedom of speech;
(3) Freedom to write for the press or to maintain a periodical publication;
(4) Freedom from arbitrary or illegal detention;
(5) Freedom of suffrage;
(6) The right against deprivation of property without due process of law;
(7) The right to a just compensation when private property is taken for public use;
(8) The right to the equal protection of the laws;
(9) The right to be secure in one's person, house, papers, and effects against unreasonable searches
and seizures;
(10) The liberty of abode and of changing the same;
(11) The privacy of communication and correspondence;
(12) The right to become a member of associations or societies for purposes not contrary to law;
(13) The right to take part in a peaceable assembly to petition the government for redress of grievances;
(14) The right to be free from involuntary servitude in any form;
(15) The right of the accused against excessive bail;
(16) The right of the accused to be heard by himself and counsel, to be informed of the nature and cause
of the accusation against him, to have a speedy and public trial, to meet the witnesses face to face, and
to have compulsory process to secure the attendance of witness in his behalf;
(17) Freedom from being compelled to be a witness against one's self, or from being forced to confess
guilt, or from being induced by a promise of immunity or reward to make such confession, except when
the person confessing becomes a State witness;
(18) Freedom from excessive fines, or cruel and unusual punishment, unless the same is imposed or
inflicted in accordance with a statute which has not been judicially declared unconstitutional; and
(19) Freedom of access to the courts.

In any of the cases referred to in this article, whether or not the defendant's act or omission constitutes a
criminal offense, the aggrieved party has a right to commence an entirely separate and distinct civil action
for damages, and for other relief. Such civil action shall proceed independently of any criminal
prosecution (if the latter be instituted), and mat be proved by a preponderance of evidence.
The indemnity shall include moral damages. Exemplary damages may also be adjudicated.
The responsibility herein set forth is not demandable from a judge unless his act or omission constitutes a
violation of the Penal Code or other penal statute.

Art. 33, Civil Code of the Philippines. In cases of defamation, fraud, and physical injuries a civil action
for damages, entirely separate and distinct from the criminal action, may be brought by the injured party.
Such civil action shall proceed independently of the criminal prosecution, and shall require only a
preponderance of evidence.

Art. 34, Civil Code of the Philippines. When a member of a city or municipal police force refuses or fails
to render aid or protection to any person in case of danger to life or property, such peace officer shall be
primarily liable for damages, and the city or municipality shall be subsidiarily responsible therefor. The
civil action herein recognized shall be independent of any criminal proceedings, and a preponderance of
evidence shall suffice to support such action.

D. Effect of death on civil actions

Section 4, Rule 111:


Effect of death on civil actions. — The death of the accused after arraignment and during the pendency of
the criminal action shall extinguish the civil liability arising from the delict. However, the independent civil
action instituted under section 3 of this Rule or which thereafter is instituted to enforce liability arising from
other sources of obligation may be continued against the estate or legal representative of the accused
after proper substitution or against said estate, as the case may be. The heirs of the accused may be
substituted for the deceased without requiring the appointment of an executor or administrator and the
court may appoint a guardian ad litem for the minor heirs.
The court shall forthwith order said legal representative or representatives to appear and be substituted
within a period of thirty (30) days from notice.
A final judgment entered in favor of the offended party shall be enforced in the manner especially
provided in these rules for prosecuting claims against the estate of the deceased.
If the accused dies before arraignment, the case shall be dismissed without prejudice to any civil action
the offended party may file against the estate of the deceased.

E. Judgment in civil action not a bar

Section 5, Rule 111:


Judgment in civil action not a bar. — A final judgment rendered in a civil action absolving the defendant
from civil liability is not a bar to a criminal action against the defendant for the same act or omission
subject of the civil action.

F. Suspension by reason of prejudicial question

Section 5, Rule 111:


Suspension by reason of prejudicial question. — A petition for suspension of the criminal action based
upon the pendency of a prejudicial question in a civil action may be filed in the office of the prosecutor or
the court conducting the preliminary investigation. When the criminal action has been filed in court for
trial, the petition to suspend shall be filed in the same criminal action at any time before the prosecution
rests.

Cases:

1.Dreamwork Construction, Inc. vs. Janiola

FACTS:
Petitioner Dreamwork Construction, through its president and VP, filed a complaint for violation of BP22
against private respondent Janiola with the Office of City Prosecutor. Janiola then instituted a civil
complaint against petitioner for the rescission of an alleged construction agreement between the parties,
as well as for damages. Notably, the checks that were subject of the criminal cases before the MTC were
issued inconsideration of the construction agreement. Janiola filed a Motion to Suspend Proceedings in
the criminal case, alleging that the civil and criminal cases involved facts and issues similar or intimately
related such that in the resolution of the issues in the civil case, the guilt or innocence of the accused
would necessarily be determined. In other words, Janiola claimed that the civil case posed a prejudicial
question as against the criminal cases.

ISSUE:
Is there a prejudicial question in this case?

RULING:
Sec. 6 Rule 111: Suspension by reason of prejudicial question. - A petition for suspension of the criminal
action based upon the pendency of a prejudicial question in a civil action may be filed in the office of the
prosecutor or the court conducting the preliminary investigation. When the criminal action has been filed
in court for trial, the petition to suspend shall be filed in the same criminal action at any time before the
prosecution rests.

SEC. 7 Rule 111: Elements of prejudicial question.


The elements of a prejudicial question are:

(a) the previously instituted civil action involves an issue similar or intimately related to the issue raised in
the subsequent criminal action, and

(b) the resolution of such issue determines whether or not the criminal action may proceed. A prejudicial
question is understood in law as that which must precede the criminal action and which requires a
decision before the final judgment can be rendered in the criminal action with which said question is
closely connected.
The civil action must be instituted prior to the institution of the criminal action.
In any event, even if the civil case here was instituted prior to the criminal action, there is, still,no
prejudicial question to speak ofthat would justify the suspension of the proceedings in the criminal case. It
must be emphasized that the gravamen of the offense charged is the issuance of a bad check. The
purpose for which the check was issued is irrelevant to the prosecution and conviction of petitioner. The
clear intention of the framers of BP 22 is to make the mere act of issuing a worthless check
malumprohibitum. It is clear that the second element required for the existence of a prejudicial question
(resolution of the issue in the civil action would determine whether the criminal action may proceed) is
absent in the instant case. Thus, no prejudicial question exists and the rules on it are inapplicable in this
case.

G. Elements of prejudicial question

Section 7, Rule 111:


Elements of prejudicial question. — The elements of a prejudicial question are: (a) the previously
instituted civil action involves an issue similar or intimately related to the issue raised in the subsequent
criminal action, and (b) the resolution of such issue determines whether or not the criminal action may
proceed.
RULE 112: PRELIMINARY INVESTIGATION

A. Preliminary Investigation defined; when required

Section 1.Preliminary investigation defined; when required. — Preliminary investigation is an inquiry


or proceeding to determine whether there is sufficient ground to engender a well-founded belief that a
crime has been committed and the respondent is probably guilty thereof, and should be held for trial.
Except as provided in section 7 of this Rule, a preliminary investigation is required to be conducted before
the filing of a complaint or information for an offense where the penalty prescribed by law is at least four
(4) years, two (2) months and one (1) day without regard to the fine.

 Nature, purpose, scope of preliminary investigation

Cases:

1. Sales vs. Sandiganbayan

FACTS:
The petitioner, the incumbent mayor of Pagudpud Ilocos Norte, shot the former mayor and his political
rival Atty. Benemerito. After the shooting, he surrendered himself and hence the police inspector and wife
of the victim filed a criminal complaint for murder against him.

The judge after conducting the preliminary examination (p.e. for brevity) found probable cause and issued
a warrant of arrest. Also after conducting the preliminary investigation (p.i. for brevity), he issued a
resolution forwarding the case to the prosecutor for appropriate action. Petitioner received a subpoena
directing him to file his counter affidavit, affidavit of witnesses and other supporting documents. He did it
the following day.

While proceedings are ongoing, he filed a petition for habeas corpus with the C.A alleging that: the
warrant was null and void because the judge who issued it was a relative by affinity of the private
respondent and the p.e. and the p.i. were illegal and irregular as the judge doesn’t have jurisdiction on the
case. The C.A. granted the petition holding that the judge was a relative by affinity by 3rd degree to the
private respondent and the p.i. he conducted has 2 stages, the p.e. and the p.i. proper. The proceeding
now consists only of one stage. He conducted the requisite investigation prior to the issuance of warrant
of arrest. Moreover, he did not complete it. He only examined the witness of the complainant. But the
prosecution instead of conducting p.i. of his own forwarded the records to the Ombudsman (OMB for
brevity) for the latter to conduct the same.

The OMB directed the petitioner to submit his counter affidavit, but he did not comply with it finding the
same superfluous. The graft investigator recommended the filing of information for murder which the OMB
approved. Petitioner received a copy of the resolution but prevented seeking reconsideration thereof he
filed a motion to defer issuance of warrant of arrest pending the determination of probable cause. The
Sandiganbayan denied the motion. This is now a petition for review on the decision of the
Sandiganbayan.

ISSUE:
Whether or Not petitioner was afforded an opportunity to be heard and to submit controverting evidence.

RULING:
The purpose of a preliminary investigation or a previous inquiry of some kind, before an accused
person is placed ontrial, is to secure the innocent against hasty, malicious and oppressive prosecution
and to protect him from an open and public accusation of acrime, from the trouble, expenses and anxiety
of a public trial. It is alsointended to protect the state from having to conduct useless andexpensive trials.
While the right is statutory rather than constitutional inits fundament, it is a component part of due process
in criminal justice.The right to have a preliminary investigation conducted before being bound over to trial
for a criminal offense and hence formally at risk of incarceration or some other penalty, is not a mere
formal or technicalright; it is a substantive right. To deny the accused’s claim to a preliminary investigation
would be to deprive him of the full measure of his right to due process

In the present case, the proper procedure in the conduct of preliminary investigation was not followed
because of the following reasons.

Firstly, the preliminary investigation was conducted by 3 different investigators, none of


whomcompleted the preliminary investigation There was not one continuous proceeding but rather, cases
of passing the buck, the last one being the OMB throwing the buck to the Sandiganbayan.

Secondly, the charge of murder is a non bailable offense. The gravity of the offense alone should have
merited a deeper and more thorough preliminary investigation. The OMB did nothing of the sort but
wallowed the resolution of the graft investigator. He did a worse job than the judge, by actually adopting
the resolution of the graft investigator without doing anything and threw everything to the Sandiganbayan
for evaluation.

Thirdly, a person under preliminary investigation by the OMB is entitled to a motion for reconsideration, as
maintained by the Rules of Procedure by the OMB. The filing of the motion for reconsideration is an
integral part of the preliminary investigation proper. The denial thereof is tantamount to the denial of the
right itself to a preliminary investigation. This fact alone renders preliminary investigation conducted in this
case incomplete.

And lastly, it was patent error for the Sandiganbayan to have relied purely on the OMB’s certification of
probable cause given the prevailing facts of the case much more so in the face of the latter’s flawed
report and one side factual findings.The court cannot accept the Sandiganbayan’s assertion of having
found probable cause on its own, considering the OMB’s defective report and findings, which merely
relied on the testimonies of the witnesses for the prosecution and disregarded the evidence for the
defense.

Judgment is rendered setting aside the resolution of the Sandiganbayan, ordering the Sandiganbayan to
quash the warrant of arrest and remanding the OMB for completion of the preliminary investigation.

2. Baytan vs. COMELEC

FACTS:
ReynatoBaytan registered as a voter in two precincts and the COMELEC En Banc affirmed the
recommendation of its Law Department to file information of double registration in violation of the Election
Code. Baytan filed with the Supreme Court a petition for certiorari on the grounds, among others, that
there was no probable cause and that election cases must first be heard and decided by a Division before
the COMELEC En Banc can assume jurisdiction.

ISSUE:
Whether or not COMELEC acted with grave abuse of discretion when it recommended the prosecution of
the petitioners despite lack of intent and substantial compliance with cancellation of double registration.

RULING:
It is well- settled that the finding of probable cause in the prosecution of election offenses rests in the
sound discretion of the COMELEC. Generally, the Court will not interfere with such finding of the
COMELEC, absent a clear showing of grave abuse of discretion. This principle emanates from the
exclusive power of the COMELEC to conduct preliminary investigation of all election investigation of all
election offenses and to prosecute the same.

Under Sec. 2, Art. IX-C of the Constitution, the COMELEC exercises both administrative and quasi-
judicial powers. The administrative powers are found in Sec 2. (1), (3) to (9) of Art IX-C. The Constitution
does not provide on whether these administrative powers shall be exercised by the COMELEC en banc
or in division. The COMELEC en banc therefore can act on administrative matters, and this had been the
practice under the 1973 and 1987 Constitutions. The prosecution by the COMELEC of violations of
election laws is an administrative power.

The exercise by the COMELEC of its quasi-judicial powers is subject to Sec.3, Art.IX-C which expressly
requires that all election cases, including pre-proclamation controversies, shall be decided by the
COMELEC in division, and the motion for reconsideration shall be decided by the COMELEC en banc.

3. Paderanga vs. Drilon

FACTS:
On 16 October 1986, an information for multiple murder was filed in the Regional Trial Court, Gingoog
City, against Felipe Galarion, Manuel Sabit, Cesar Sabit, Julito Ampo, Eddie Torion, John Doe, Peter Doe
and Richard Doe, for the deaths on 1 May 1984 of Renato Bucag, his wife Melchora Bucag, and their son
Renato Bucag II. Venue was, however, transferred to Cagayan de Oro City per Administrative Matter 87-
2-244. Only Felipe Galarion was tried and found guilty as charged. The rest of the accused remained at
large. Felipe Galarion, however, escaped from detention and has not been apprehended since then. In an
amended information filed on 6 October 1988, Felizardo Roxas, alias "Ely Roxas," "Fely Roxas" and
"Lolong Roxas," was included as a co-accused. Roxas retained Atty. Miguel P. Paderanga as his counsel.
As counsel for Roxas, Paderanga filed, among others, an Omnibus Motion to dismiss, to Quash the
Warrant of Arrest and to Nullify the Arraignment on 14 October 1988.

The trial court in an order dated 9 January 1989, denied the omnibus motion but directed the City
Prosecutor to conduct another preliminary investigation or reinvestigation in order to grant the accused
all the opportunity to adduce whatever evidence he has in support of his defense. In the course of the
preliminary investigation, through a signed affidavit, Felizardo Roxas implicated Atty. Paderanga in the
commission of the crime charged. The City Prosecutor of Cagayan de Oro City inhibited himself from
further conducting the preliminary investigation against Paderanga at the instance of the latter's counsel,
per his resolution dated 7 July 1989. In his first indorsement to the Department of Justice, dated 24 July
1989, said city prosecutor requested the Department of Justice to designate a state prosecutor to
continue the preliminary investigation against Paderanga.

In a resolution dated 6 September 1989, the State Prosecutor Henrick F. Gingoyon, who was designated
to continue with the conduct of the preliminary investigation against Paderanga, directed the amendment
of the previously amended information to include and implead Paderanga as one of the accused therein.
Paderanga moved for reconsideration, contending that the preliminary investigation was not yet
completed when said resolution was promulgated, and that he was deprived of his right to present a
corresponding counter-affidavit and additional evidence crucial to the determination of his alleged
"linkage" to the crime charged. The motion was, however, denied by Gingoyon in his order dated 29
January 1990.

From the aforesaid resolution and order, Paderanga filed a Petition for Review with the Department of
Justice. Thereafter, he submitted a Supplemental Petition with Memorandum, and then a Supplemental
Memorandum with Additional Exculpatory/Exonerating Evidence Annexed, attaching thereto an affidavit
of Roxas dated 20 June 1990 and purporting to be a retraction of his affidavit of 30 March 1990 wherein
he implicated Paderanga. On 10 August 1990, the Department of Justice, through Undersecretary
Silvestre H. Bello III, issued Resolution 648 dismissing the said petition for review. His motion for
reconsideration having been likewise denied, Paderanga then filed the petition for mandamus and
prohibition before the Supreme Court.

ISSUE:
Whether there is no prima facie evidence, or probable cause, or sufficient justification to hold Paderanga
to a tedious and prolonged public trial.

RULING:
A preliminary investigation is defined as an inquiry or proceeding for the purpose of determining whether
there is sufficient ground to engender a well founded belief that a crime cognizable by the Regional Trial
Court has been committed and that the respondent is probably guilty thereof, and should be held for trial.
The quantum of evidence now required in preliminary investigation is such evidence sufficient to
"engender a well founded belief" as to the fact of the commission of a crime and the respondent's
probable guilt thereof.

A preliminary investigation is not the occasion for the full and exhaustive display of the parties' evidence;
it is for the presentation of such evidence only as may engender a well grounded belief that an offense
has been committed and that the accused is probably guilty thereof. Preliminary investigation is generally
inquisitorial, and it is often the only means of discovering the persons who may be reasonably charged
with a crime, to enable the fiscal to prepare his complaint or information. It is not a trial of the case on the
merits and has no purpose except that of determining whether a crime has been committed and whether
there is probable cause to believe that the accused is guilty thereof, and it does not place the person
against whom it is taken in jeopardy. The institution of a criminal action depends upon the sound
discretion of the fiscal. He has the quasi-judicial discretion to determine whether or not a criminal case
should be filed in court. Hence, the general rule is that an injunction will not be granted to restrain a
criminal prosecution.

The case of Brocka, et al. vs. Enrile, et al. cites several exceptions to the rule, to wit: (a) To afford
adequate protection to the constitutional rights of the accused; (b) When necessary for the orderly
administration of justice or to avoid oppression or multiplicity of actions; (c) When there is a prejudicial
question which is sub-judice; (d) When the acts of the officer are without or in excess of authority; (e)
Where the prosecution is under an invalid law, ordinance or regulation; (f) When double jeopardy is
clearly apparent; (g) Where the court has no jurisdiction over the offense; (h) Where it is a case of
persecution rather than prosecution; (i) Where the charges are manifestly false and motivated by the lust
for vengeance; and (j) When there is clearly no prima facie case against the accused and a motion to
quash on that ground has been denied. A careful analysis of the circumstances obtaining in the present
case, however, will readily show that the same does not fall under any of the aforesaid exceptions.

4. Go vs. CA

FACTS:
Petitioner, while traveling in the wrong direction on a one-way street, almost had a collision with another
vehicle. Petitioner thereafter got out of his car, shot the driver of the other vehicle, and drove off. An
eyewitness of the incident was able to take down petitioner’s plate number and reported the same to the
police, who subsequently ordered a manhunt for petitioner. 6 days after the shooting, petitioner presented
himself in the police station, accompanied by 2 lawyers, the police detained him. Subsequently a criminal
charge was brought against him.

Petitioner posted bail, while the prosecutor filed the case to the lower court, setting and commencing trial
without preliminary investigation. Prosecutor reasons that the petitioner has waived his right to preliminary
investigation as bail has been posted and that such situation, that petitioner has been arrested without a
warrant lawfully, falls under Section 5, Rule 113 and Section 7, Rule 112 of The 1985 Rules of Criminal
Procedure which provides for the rules and procedure pertaining to situations of lawful warrantless
arrests. Petitioner in his petition for certiorari assails such procedure and actions undertaken and files for
a preliminary investigation.

ISSUE:
Whether or Not petitioner effectively waived his right to preliminary investigation.

RULING:
Petitioner and prosecutor err in relying on Umil v. Ramos, wherein the Court upheld the warrantless arrest
as valid effected 1 to 14 days from actual commission of the offenses, which however constituted
“continuing crimes,” i.e. subversion, membership in an outlawed organization, etc. There was no lawful
warrantless arrest under Section 5, Rule 113. This is because the arresting officers were not actually
there during the incident, thus they had no personal knowledge and their information regarding petitioner
were derived from other sources. Further, Section 7, Rule 112, does not apply.
Petitioner was not arrested at all, as when he walked in the police station, he neither expressed surrender
nor any statement that he was or was not guilty of any crime. When a complaint was filed to the
prosecutor, preliminary investigation should have been scheduled to determine probable cause.
Prosecutor made a substantive error, petitioner is entitled to preliminary investigation, necessarily in a
criminal charge, where the same is required appear thereat. Petition granted, prosecutor is ordered to
conduct preliminary investigation, trial for the criminal case is suspended pending result from preliminary
investigation, petitioner is ordered released upon posting a bail bond.

5. Doromal vs. Sandiganbayan

FACTS:
Quintin S. Doromal, a former Commissioner of the Presidential Commission on Good Government
(PCGG), for violation of the Anti-Graft and Corrupt Practices Act (RA 3019), Sec. 3(h), in connection with
his shareholdings and position as president and director of the Doromal International Trading Corporation
(DITC) which submitted bids to supply P61 million worth of electronic, electrical, automotive, mechanical
and airconditioning equipment to the Department of Education, Culture and Sports (or DECS) and the
National Manpower and Youth Council (or NMYC).

An information was then filed by the “Tanodbayan” against Doromal for the said violation and a
preliminary investigation was conducted.

The petitioner then filed a petition for certiorari and prohibition questioning the jurisdiction of the
“Tanodbayan” to file the information without the approval of the Ombudsman.

The Supreme Court held that the incumbent Tanodbayan (called Special Prosecutor under the 1987
Constitution and who is supposed to retain powers and duties NOT GIVEN to the Ombudsman) is clearly
without authority to conduct preliminary investigations and to direct the filing of criminal cases with the
Sandiganbayan, except upon orders of the Ombudsman. Subsequently annulling the information filed by
the “Tanodbayan”.

A new information, duly approved by the Ombudsman, was filed in the Sandiganbayan, alleging that the
Doromal, a public officer, being then a Commissioner of the Presidential Commission on Good
Government, did then and there wilfully and unlawfully, participate in a business through the Doromal
International Trading Corporation, a family corporation of which he is the President, and which company
participated in the biddings conducted by the Department of Education, Culture and Sports and the
National Manpower & Youth Council, which act or participation is prohibited by law and the constitution.
The petitioner filed a motion to quash the information on the ground that it was invalid since there had
been no preliminary investigation for the new information that was filed against him.

The motion was denied by Sandiganbayan claiming that another preliminary investigation is unnecessary
because both old and new informations involve the same subject matter.

ISSUES:
1. Whether or not preliminary investigation is necessary even if both informations involve the same
subject matter.

2. Whether or not the information shall be effected as invalid due to the absence of preliminary
investigation.

RULING:
Yes, as to the first issue. No, as to the second issue. Petition was granted by the Supreme Court.

1. The right of the accused to a preliminary investigation is "a substantial one." Its denial over his
opposition is a "prejudicial error, in that it subjects the accused to the loss of life, liberty, or property
without due process of law" provided by the Constitution.
Since the first information was annulled, the preliminary investigation conducted at that time shall also be
considered as void. Due to that fact, a new preliminary investigation must be conducted.

2. The absence of preliminary investigation does not affect the court's jurisdiction over the case. Nor do
they impair the validity of the information or otherwise render it defective; but, if there were no preliminary
investigations and the defendants, before entering their plea, invite the attention of the court to their
absence, the court, instead of dismissing the information should conduct such investigation, order the
fiscal to conduct it or remand the case to the inferior court so that the preliminary investigation may be
conducted.

WHEREFORE, the petition for certiorari and prohibition is granted. The Sandiganbayan shall immediately
remand Criminal Case No. 12893 to the Office of the Ombudsman for preliminary investigation and shall
hold in abeyance the proceedings before it pending the result of such investigation.

6. Webb vs. De Leon

FACTS:
On June 19, 1994, the National Bureau of Investigation filed with the DOJ a letter-complaint charging
petitioners Hubert Webb, Michael Gatchalian, Antonio J. Lejano and 6 other persons with the crime of
Rape and Homicide of Carmela N. Vizconde, her mother Estrellita Nicolas-Vizconde, and her sister Anne
Marie Jennifer in their home at Number 80 W. Vinzons, St., BF Homes, Paranaque, Metro Manila on June
30, 1991.

Forthwith, the DOJ formed a panel of prosecutors headed by Asst Chief State Prosecutor Jovencio R.
Zuno to conduct the preliminary investigation.

Petitioners fault the DOJ Panel for its finding of probable cause. They assail the credibility of Jessica
Alfaro as inherently weak and uncorroborated due to her inconsistencies between her April 28, 1995 and
May 22, 1995 sown statements. They criticize the procedure followed by the DOJ Panel when it did not
examine witnesses to clarify the alleged inconsistencies. They also charge that respondent Judge Raul
de Leon and respondent Judge Amelita Tolentino issued warrants of arrest against them without
conducting the required preliminary examination, and complain about the denial of their constitutional
right to due process and violation of their right to an impartial investigation. They also assail the
prejudicial publicity that attended their preliminary investigation.

ISSUES:
1. Did respondent judges de Leon and Tolentino gravely abuse their discretion when they failed to
conduct a preliminary examination before issuing warrants of arrest against the accused?
2. Did the DOJ Panel deny them their constitutional right to due process during their preliminary
investigation?

RULING:
1. NO. In arrest cases, there must be a probable cause that a crime has been committed and that the
person arrested committed it.
Section 6 of Rule 112 provides that – “upon filing of an information, the RTC may issue a warrant for the
accused.”

Clearly then, our laws repudiate the submission that respondent judges should have conducted
“searching examination of witnesses” before issuing warrants of arrest against them.

2. No. There is no merit in this contention because petitioners were given all the opportunities to be
heard.
The DOJ Panel precisely requested the parties to adduce more evidence in their behalf and for the panel
to study the evidence submitted more fully.
 Probable Cause

Cases:

1. Allado vs. Diokno

FACTS:
The crime alleged to petitioner Allado and Mendoza, by the PACC[Presidential Anti-Crime Commission],
is heinous crime of kidnapping with murder of Eugen Alexander Van Twest, a German national, whois
reportedly an international fugitive from justice.

Other incidental crimes charged were illegal possession of firearms and ammunition, carnapping, and
usurpation of authority.

The whole investigation was triggered by an extrajudicial confessionby a Security Guard Escolastico
Umbal, a discharge of the PhilippineConstabulary.
Based on the confession of Umbal, Judge Barrios of RTCBr 11 issued a search warrant against
petitioners.

For the institution of criminal proceedings the DOJ took over thecase, after a new panel of prosecutors
were recommended and after preliminary investigation found probable cause that accusedcommitted the
crime and submitted the case for trial which was assigned to Judge Diokno of Br 62. Judge Diokno after
finding probable cause issued warrant of arrest.

ISSUE:
Whether or not the judge erred in finding probable cause issuing the search warrant.

RULING:
Yes, there is no probable cause in this case. The probable cause test is an objective one, for in order that
there be probable cause the facts and circumstances must be such as would warrant a belief by a
reasonably discreet and prudent man that the accused is guilty of the crime which has just been
committed. There is serious doubt on Van Twest’s reported death since the corpus delicti has not been
established, nor have his remains been recovered. We are reminded of the leading case of U.S. v.
Samarin decided ninety-two years ago where this Court ruled that when the supposed victim is wholly
unknown, his body not found, and there is but one witness who testifies to the killing, the corpus delicti is
not sufficiently proved. In People v. Inting, we emphasized the important features of the constitutional
mandate:

(a) The determination of probable cause is a function of the judge; it is not for the provincial fiscal or
prosecutor to ascertain. Only the judge and the judge alone makes this determination;

(b) The preliminary inquiry made by a prosecutor does not bind the judge. It merely assists him in making
the determination of probable cause. The judge does not have to follow what the prosecutor presents to
him. By itself, the prosecutor’s certification of probable cause is ineffectual. It is the report, the affidavits,
the transcript of stenographic notes (if any), and all other supporting documents behind the prosecutor’s
certification which are material in assisting the judge in his determination of probable cause; and

(c) Judges and prosecutors alike should distinguish the preliminary inquiry which determines probable
cause for the issuance of a warrant of arrest from the preliminary investigation proper which ascertains
whether the offender should be held for trial or released.

Even if the two inquiries be conducted in the course of one and the same proceeding, there should be no
confusion about their objectives. The determination of probable cause for the warrant is made by the
judge. The preliminary investigation proper whether or not there is reasonable ground to believe that the
accused is guilty of the offense charged and therefore, whether or not he should be subjected to the
expense, rigors and embarrassment of trial is a function of the prosecutor.
The extrajudicial statement of Umbal suffers from material inconsistencies. In Lim v. Felix, where we
reiterated Soliven v. Makasiar and People v. Inting , we said -

The Judge does not have to personally examine the complainant and his witnesses. The Prosecutor can
perform the same functions as a commissioner for the taking of the evidence. However, there should be a
report and necessary documents supporting the Fiscal’s bare certification. All these should be before the
Judge. The extent of the Judge’s personal examination of the report and its annexes depends on the
circumstances of each case. We cannot determine beforehand how cursory or exhaustive the
Judge’s examination should be.

The Judge has to exercise sound discretion for, after all, the personal determination is vested in the
Judge by the Constitution. It can be as brief or as detailed as the circumstances of each case require. To
be sure, the judge must go beyond the Prosecutor’s certification and investigation report whenever
necessary. He should call for the complainant and witnesses themselves to answer the court’s probing
questions when the circumstances of the case so require.

B. Officers authorized to conduct preliminary investigations

1. DOJ-NPS Manual, PART III. PRELIMINARY INVESTIGATION

SEC. 6. Officers Authorized to Conduct Preliminary Investigation. - The following may conduct a
preliminary investigation;1
a) Provincial or City Prosecutors and their assistants;
b) Judges of Municipal Trial Courts and Municipal Circuit Trial Courts;
c) National and Regional State Prosecutors; and
d) Other officers as may be authorized by law.

2. RA 6770

Section 11. Structural Organization. — The authority and responsibility for the exercise of the mandate
of the Office of the Ombudsman and for the discharge of its powers and functions shall be vested in the
Ombudsman, who shall have supervision and control of the said office.

(4) The Office of the Special Prosecutor shall, under the supervision and control and upon the
authority of the Ombudsman, have the following powers:

(a) To conduct preliminary investigation and prosecute criminal cases within the
jurisdiction of the Sandiganbayan.

Section 15. Powers, Functions and Duties. — The Office of the Ombudsman shall have the following
powers, functions and duties:
(1) Investigate and prosecute on its own or on complaint by any person, any act or omission of any public
officer or employee, office or agency, when such act or omission appears to be illegal, unjust, improper or
inefficient.t has primary jurisdiction over cases cognizable by the Sandiganbayan and, in the exercise of
this primary jurisdiction, it may take over, at any stage, from any investigatory agency of Government, the
investigation of such cases;

(2) Direct, upon complaint or at its own instance, any officer or employee of the Government, or of any
subdivision, agency or instrumentality thereof, as well as any government-owned or controlled
corporations with original charter, to perform and expedite any act or duty required by law, or to stop,
prevent, and correct any abuse or impropriety in the performance of duties;

(3) Direct the officer concerned to take appropriate action against a public officer or employee at fault or
who neglect to perform an act or discharge a duty required by law, and recommend his removal,
suspension, demotion, fine, censure, or prosecution, and ensure compliance therewith; or enforce its
disciplinary authority as provided in Section 21 of this Act: provided, that the refusal by any officer without
just cause to comply with an order of the Ombudsman to remove, suspend, demote, fine, censure, or
prosecute an officer or employee who is at fault or who neglects to perform an act or discharge a duty
required by law shall be a ground for disciplinary action against said officer;

(4) Direct the officer concerned, in any appropriate case, and subject to such limitations as it may provide
in its rules of procedure, to furnish it with copies of documents relating to contracts or transactions
entered into by his office involving the disbursement or use of public funds or properties, and report any
irregularity to the Commission on Audit for appropriate action;

(5) Request any government agency for assistance and information necessary in the discharge of its
responsibilities, and to examine, if necessary, pertinent records and documents;

(6) Publicize matters covered by its investigation of the matters mentioned in paragraphs (1), (2), (3) and
(4) hereof, when circumstances so warrant and with due prudence: provided, that the Ombudsman under
its rules and regulations may determine what cases may not be made public: provided, further, that any
publicity issued by the Ombudsman shall be balanced, fair and true;

(7) Determine the causes of inefficiency, red tape, mismanagement, fraud, and corruption in the
Government, and make recommendations for their elimination and the observance of high standards of
ethics and efficiency;

(8) Administer oaths, issue subpoena and subpoena duces tecum, and take testimony in any
investigation or inquiry, including the power to examine and have access to bank accounts and records;

(9) Punish for contempt in accordance with the Rules of Court and under the same procedure and with
the same penalties provided therein;

(10) Delegate to the Deputies, or its investigators or representatives such authority or duty as shall
ensure the effective exercise or performance of the powers, functions, and duties herein or hereinafter
provided;

(11) Investigate and initiate the proper action for the recovery of ill-gotten and/or unexplained wealth
amassed after February 25, 1986 and the prosecution of the parties involved therein.

The Ombudsman shall give priority to complaints filed against high ranking government officials and/or
those occupying supervisory positions, complaints involving grave offenses as well as complaints
involving large sums of money and/or properties.

Cases:

3. Velasco vs. Casaclang

FACTS:
By virtue of Assignment Order No. 89-846 dated June 5, 1989 of the Commission on Audit, COA Audit
Examiners Priscilla G. Cruz and Virginia G. Pantoja conducted a special audit of selected transactions of
the Armed Forces of the Philippines (AFP) Logistics Command, covering the period from January 1988 to
May 1989.
In their Memorandum Report, dated March 13, 1990, to the Chairman of the Commission on Audit, the
said COA audit examiners reported their findings.

On May 6, 1993, the same audit examiners filed with the Office of the Ombudsman a Joint Affidavit-
Complaint deploring the aforesaid transactions dubbed as anomalous and highly irregular.
On May 27, 1993, respondent Deputy Ombudsman for the Military issued an Order in OMB Case No. 0-
90-0296, entitled Commission on Audit, et al. vs. BGen. Buenaventura Tabo, et al., to wit:

It appearing from the affidavit-complaint dated 6 May 1993 filed by complainant/s to be sufficient in form
and substance, respondent/s is/are hereby directed to file within ten (10) days from receipt hereof
his/her/their counter-affidavit/s and other controverting evidence with proof of service upon complainant/s
who may file his/her/their reply/ies within ten (10) days from receipt of the same. Failure of the
respondent/s to do so shall be construed as a waiver of his/her/their right/s to be heard and the
preliminary investigation of this case shall proceed accordingly, thereafter, the same shall be deemed
submitted for resolution.

Petitioner Laura Velasco questioned the authority of respondent Deputy Ombudsman to conduct
preliminary investigation, arguing that pursuant to Section 11, par. (4), subpar. (a) of R.A. 6770, the Office
of the Special Prosecutor is vested with the power and authority to conduct preliminary investigation and
to prosecute criminal cases falling within the jurisdiction of the Sandiganbayan and Section 17 of P.D.
1630 provides that the Office of Tanodbayan (now Office of the Special Prosecutor) has the exclusive
authority to conduct preliminary investigation in all cases cognizable by the Sandiganbayan.

ISSUE:
Whether or not respondent Deputy Ombudsman has authority to conduct preliminary investigation.

RULING:
Under Section 2, Rule 112 of the 1985 Rules of Criminal Procedure, as amended:
The following may conduct preliminary investigation:

(a) Provincial or city fiscals and their assistants;


(b) Judges of the Municipal Trial Courts and Municipal Circuit Trial Courts;
(c) National and Regional state prosecutors; and
(d) Such other officers as may be authorized by law.

On the other hand, Section 15, paragraph (1) of R.A. 6770, otherwise known as The Ombudsman
Act, provides:

The Office of the Ombudsman shall have the following powers, functions, and duties:

(1) Investigate and prosecute on its own or on complaint by any person, any act or omission of any public
officer or employee, office, or agency, when such act or omission appears to be illegal, unjust, improper
or inefficient. It has a primary jurisdiction over cases cognizable by the Sandiganbayan and, in the
exercise of this primary jurisdiction, it may take over, at any stage, from any investigatory agency of
Government, the investigation of such cases.

In light of the afore quoted provisions of law in point, it is beyond cavil that the Ombudsman and his
Deputies are, within legal contemplation, other officers authorized by law to conduct preliminary
investigation.
4. Balgos vs. Sandiganbayan

FACTS:
Petitioners were charged with violation of Section 3(c) of Republic Act No. 3019, otherwise known as the
Anti-Graft and Corrupt Practice Act, as amended, in an information that was filed with the Sandiganbayan
by the Special Prosecutor which was approved by the Deputy Tanodbayan, after a preliminary
investigation.

Antonio Uy Lim, the plaintiff and prevailing party in Civil Case No. 4047 filed a complaint for rescission of
the sale of the car by Juanito Ang to private respondent Leticia Acosta-Ang for being allegedly in fraud of
creditors. The said complaint was filed with the Regional Trial Court of Nueva Vizcaya and was docketed
as Civil Case No. 5307. On the same day, petitioners filed a motion for reinvestigation in the Tanodbayan.
The same was granted on May 18,1987.

After conducting the reinvestigation, the Tanodbayan issued an order resolving to:
(a) set aside and render without force and effect its Resolution in this case dated March 25,1986;
(b) to dismiss the case for lack of merit.
(c) to withdraw the Information filed in Criminal Case No. 11414 as soon as possible in the interest of
justice.

The Tanodbayan filed with the Sandiganbayan a motion to withdraw the information against petitioners.
This was denied on June 29, 1988. On September 1, 1988, petitioners filed a motion to suspend
proceedings in the criminal case against them on the ground of the existence of a prejudicial question in
Civil Case No. 5307. This was likewise denied by the Sandiganbayan on October 24,1988.
Hence, the instant petition where it is alleged that the Sandiganbayan committed a grave abuse of
discretion amounting to lack or excess of jurisdiction in denying the aforestated motions.

ISSUE:
Whether or not the Sandiganbayan committed a grave abuse of discretion amounting to lack or excess of
jurisdiction in denying the aforestated motions.

RULING:
In the case of Crespo vs. Mogul, this Court laid down the ground rules and the parameters pertaining to
the direction and control of the prosecution of a criminal action by the fiscal or government prosecutor as
provided for in the rules 4 in relation to the jurisdiction of the competent courts over such cases. We ruled
that while the public prosecutor has the sole direction and control in the prosecution of offenses, once the
complaint or information is filed in court, the court thereby acquires jurisdiction over the case and all
subsequent actions that may be taken by the public prosecutor in relation to the disposition of the case
must be subject to the approval of the said court.

In such an instance, before a re-investigation of the case may be conducted by the public prosecutor, the
permission or consent of the court must be secured. And if after such reinvestigation the prosecution finds
a cogent basis to withdraw the information or otherwise cause the dismissal of the case, such proposed
course of action must be addressed to the sound discretion of the court.

In the past, a government prosecutor could practically impose his judgment or opinion on the court as it
was recognized that the prosecution of offenses is his exclusive domain which resulted then and again in
a clash or conflict of opinion between the prosecutors and the courts to the detriment of the administration
of justice. Such a situation may no longer be possible since Crespo. It is the court that has now the final
say on any subsequent disposition or action once the case is brought before it.
5. Alonzo vs. Concepcion

FACTS:
Jose Alonzo filed a complaint for murder against Salamat, Rances, Santos, SPO4 Alonzo and a certain
Isidro Atienza.

A preliminary investigation was conducted by the Assistant Provincial Prosecutor where Jose Alonzo and
his four witnesses testified. Upon review of the records of the case by the 3rd Assistant Provincial
Prosecutor, it was recommended that Salamat be charged with murder as principal, and Santos and
Rances as accessories. With regard to SPO4 Alonzo and Isidro Atienza, the prosecutor found that no
sufficient evidence was adduced to establish their conspiracy with Salamat. Thereafter, under the
direction of the Officer-in-Charge, an Information was prepared, charging Salamat as principal, and
Rances and Santos as accessories, for the murder of Pedrito. No bail was recommended. The case was
docketed as Criminal Case No. 4767-M-2003 with Branch 12 of the Regional Trial Court of Malolos City,
Bulacan, under presiding judge Crisanto C. Concepcion.

On December 17, 2003, Judge Concepcion issued an Order, where he directed to amend the information,
so as to include all the aforenamed persons as accused in this case, all as principals.

ISSSUE:
Whether or not the court has authority to review and reverse the resolution of the Office of the Provincial
Prosecutor or to find probable cause against a respondent for the purpose of amending the Information.

RULING:
As a rule, courts cannot interfere with the prosecutor's discretion and control of the criminal prosecution.
The reason for placing the criminal prosecution under the direction and control of the fiscal is to prevent
malicious or unfounded prosecution by private persons. However, while prosecuting officers have the
authority to prosecute persons shown to be guilty of a crime, they have equally the legal duty not to
prosecute when after an investigation, the evidence adduced is not sufficient to establish a prima
facie case. Judges should not unduly interfere with the exercise of the power to prosecute on the part of
fiscals.

It is not a sufficient excuse for respondent to aver that he did not impose any sanction for non-compliance
with his Order. In itself, his Order does violence to the principle of separation of powers enshrined in our
Constitution. In a clash of views between the judge who did not investigate and the prosecutor who did, or
between the fiscal and the offended party or the accused, that of the prosecutor's should normally
prevail.

The impact of respondent Judge's orders is that his judgment is to be substituted for that of the
prosecutor's on the matter of what crime is to be filed in court. The question of instituting a criminal
charge is one addressed to the sound discretion of the investigating Fiscal. The information he lodges in
court must have to be supported by facts brought about by an inquiry made by him. It stands to reason
then to say that in a clash of views between the judge who did not investigate and the fiscal who did, or
between the fiscal and the offended party or the defendant, those of the Fiscal's should normally prevail.
In this regard, he cannot ordinarily be subject to dictation.

We are not to be understood as saying that criminal prosecution may not be blocked in exceptional
cases. A relief in equity may be availed of to stop a purported enforcement of a criminal law where it is
necessary:

(a) for the orderly administration of justice;


(b) to prevent the use of the strong arm of the law in an oppressive and vindictive manner;
(c) to avoid multiplicity of actions;
(d) to afford adequate protection to constitutional rights; and
(e) in proper cases, because the statute relied upon is unconstitutional or was held invalid.

We understand respondents zeal in trying to uphold the ends of justice. However, respondent overlooked
the fact that there is a remedy where a prosecutor errs in not charging a person in an Information. The
recourse is to appeal to the Secretary of Justice.29 By ordering the prosecutor to include complainant,
Rances and Santos as principals in the Information, respondent arrogated unto himself the executive
power of supervision and control over public prosecutors. His conduct is not only unbecoming of a judge;
more importantly, it transgresses our Constitution.

C. Procedure

 In cases cognizable by RTC; conducted by prosecutor

Cases:

1. Rodil vs. Garcia

FACTS:
Counsel for petitioner Reynaldo Rodil, against whom a warrant of arrest had been issued the charge
against him being that of murder, insisted to recall witnesses for the prosecution to enable such counsel
to cross-examine them, on, to quote his words, "clarificatory and amplificatory matters". The same was
denied by Judge Segundo M. Garcia

The denial thereof by respondent Municipal Judge Segundo M. Garcia of Sta. Cruz, Marinduque led to
this proceeding for certiorari and prohibition with preliminary injunction.

The Petitioner files a petition praying that the order denying counsel's request to recall government
witnesses be set aside and nullified, and that bail be granted petitioner. A petition to that effect was
denied and a subsequent motion for reconsideration, still undecided.

Respondents were required to comment and the Court likewise issued a temporary restraining order.
Such a comment was submitted on behalf of respondent by the Solicitor General seeking the dismissal of
the petition on the ground that the right to cross-examine in a preliminary investigation is not a right
granted an accused and that the exercise of discretion by respondent Judge considering the evidence of
record sufficed to justify denial of the application for bail.

ISSUE:
Whether or not the right to cross-examine is guaranteed to an accused at the stage of preliminary
investigation.

RULING:
The has Court held in number of cases that at the outset, it should be stated that the refusal of the Justice
of the Peace to allow the defense to cross-examine the prosecution's witnesses presented prior to
petitioner's arrest, cannot be utilized as argument for the contention that the prosecution should not have
been allowed to cross-examine the defense witnesses.

As first set forth in the Dequito case: "There are an infinite number of things which a party may not in strict
law do or cause to be done but which may be permitted by the court in the exercise of its discretion and in
the interest of justice. Specially is this true in matters affecting the conduct of the trial and the calling,
recalling and examination of witnesses."

There is added emphasis as shown by this excerpt: "The judge is not a ministerial officer reduced to
recording what takes place and what witnesses day in the examination. Above all, his is the great
responsibility of safeguarding the accused from groundless or vindictive prosecution. If the justice of the
peace is to ascertain, as he must, whether a crime has been committed and, if so, whether there is
probable cause that the accused committed it, his authority cannot be confined as in a straight jacket to
the stiffness of medieval and outmoded technicalities of practice."

It thus appears clearly that in the exercise of his discretion respondent Judge could have granted the
request and thus avoided the necessity of this character having to be filed.

The interest of a more speedy and a more efficient administration of justice would be best served if there
is a greater awareness on the part of judges that in addition to safeguarding the express rights of an
accused person, a matter mandated by the Constitution or the Rules of Court, they should likewise
exercise their discretion in such a way that the purpose of a preliminary investigation, the avoidance of
groundless or vindictive prosecutions, could be attained in as fair and objective manner as possible

 In cases cognizable by the MTC; conducted by prosecutor

1. A. M. No. 05-8-26-SC
Section 5. When warrant of arrest may issue.
(b) By the Municipal Trial Court. —When required pursuant to the second paragraph of section 1 of this
Rule, the preliminary investigation of cases falling under the original jurisdiction of the Metropolitan Trial
Court, Municipal Trial Court in Cities, Municipal Trial Court or Municipal Circuit Trial Court shall be
conducted by the prosecutor. The procedure for the issuance of a warrant of arrest by the judge shall be
governed by paragraph (a) of this section.

2. Conquila vs. MTC Judge Bernardo

FACTS:
In a verified complaint dated 30 July 2008, complainant Conquilla charged respondent judge with
usurpation of authority, grave misconduct, and gross ignorance of the law.

Complainant alleged that on 4 July 2008, a criminal complaint for direct assault was filed against her
before the MTC of Bocaue, Bulacan. The complaint was signed by Police Chief
Inspector Rizalino Andaya of the Bocaue Police Station.

On 8 July 2008, respondent judge conducted a preliminary investigation and found probable cause to
hold the complainant for trial for the crime of direct assault. Respondent judge then issued a warrant of
arrest dated 8 July 2008, with the bail fixed at P12,000.
On 10 July 2008, upon motion of complainant, respondent judge issued an order reducing the bail for
complainants provisional liberty to P6,000. On the same date, complainant posted cash bail of P6,000 for
her provisional liberty.

Complainant then filed an administrative complaint, alleging that under A.M. No. 05-08-[2]6-SC, first level
court judges no longer have the authority to conduct preliminary investigations. Thus, complainant avers
that respondent judge committed an illegal act constituting gross ignorance of the law and procedure
when he conducted the preliminary investigation and issued the warrant of arrest.

ISSUE:
Whether or not the conduct of the respondent judge is a direct contravention of A.M. No. 05-8-26-SC?

RULING:
Yes. The Resolution in A.M. No. 05-8-26-SC, which took effect on 3 October 2005, removed the conduct
of investigation from the scope of authority of first level courts judges. Under Section 2 of Rule112, only
the following officers are authorized to conduct preliminary investigations: (a) Provincial or City
Prosecutors and their assistants; (b) National and Regional State Prosecutors; and (c) Other officers as
may be authorized by law.

Furthermore, Sec 5b of Rule 112 provides: SEC. 5. When warrant of arrest may issue. ‒
(b) By the Municipal Trial Court. ‒ When required pursuant to the second paragraph of section 1 of this
Rule, the preliminary investigation of cases falling under the original jurisdiction of the Metropolitan Trial
Court, Municipal Trial Court in Cities, Municipal Trial Court or Municipal Circuit Trial Court shall be
conducted by the prosecutor. The procedure for the issuance of a warrant of arrest by the judge shall be
governed by paragraph (a) of this section.

MTC judges are no longer authorized to conduct preliminary investigation. The complaint was direct
assault a public school teacher. The duration of the penalty of prision correccional in its medium and
maximum periods is 2 years, 4 months and 1 day to 6 years.

Thus, the offense charged against complainant requires the conduct of preliminary investigation as
provided under Section 1 of Rule 112of the Rules of Court. It was therefore incumbent upon respondent
judge to forward the records of the case to the Office of the Provincial Prosecutor for preliminary
investigation, instead of conducting the preliminary investigation himself.

In this case, respondent judge conducted the preliminary investigation without authority and issued the
warrant of arrest. Thus, these acts are void for want of jurisdiction. The reduction of bail is also void
because in the first place, respondent judge had no jurisdiction over the case itself.

 In cases cognizable by the Sandiganbayan

1. Conducted by prosecutor; Sec. 3, 4, 5, 8, Rule 112

RULE 112. Preliminary Investigation

Section 3. Procedure. — The preliminary investigation shall be conducted in the following manner:
(a) The complaint shall state the address of the respondent and shall be accompanied by the affidavits of
the complainant and his witnesses, as well as other supporting documents to establish probable cause.
They shall be in such number of copies as there are respondents, plus two (2) copies for the official file.
The affidavits shall be subscribed and sworn to before any prosecutor or government official authorized to
administer oath, or, in their absence or unavailability, before a notary public, each of who must certify that
he personally examined the affiants and that he is satisfied that they voluntarily executed and understood
their affidavits.

(b) Within ten (10) days after the filing of the complaint, the investigating officer shall either dismiss it if he
finds no ground to continue with the investigation, or issue a subpoena to the respondent attaching to it a
copy of the complaint and its supporting affidavits and documents.

The respondent shall have the right to examine the evidence submitted by the complainant which he may
not have been furnished and to copy them at his expense. If the evidence is voluminous, the complainant
may be required to specify those which he intends to present against the respondent, and these shall be
made available for examination or copying by the respondent at his expense.

Objects as evidence need not be furnished a party but shall be made available for examination, copying,
or photographing at the expense of the requesting party.

(c) Within ten (10) days from receipt of the subpoena with the complaint and supporting affidavits and
documents, the respondent shall submit his counter-affidavit and that of his witnesses and other
supporting documents relied upon for his defense. The counter-affidavits shall be subscribed and sworn
to and certified as provided in paragraph (a) of this section, with copies thereof furnished by him to the
complainant. The respondent shall not be allowed to file a motion to dismiss in lieu of a counter-affidavit.

(d) If the respondent cannot be subpoenaed, or if subpoenaed, does not submit counter-affidavits within
the ten (10) day period, the investigating officer shall resolve the complaint based on the evidence
presented by the complainant.
(e) The investigating officer may set a hearing if there are facts and issues to be clarified from a party or a
witness. The parties can be present at the hearing but without the right to examine or cross-examine.
They may, however, submit to the investigating officer questions which may be asked to the party or
witness concerned.
The hearing shall be held within ten (10) days from submission of the counter-affidavits and other
documents or from the expiration of the period for their submission. It shall be terminated within five (5)
days.

(f) Within ten (10) days after the investigation, the investigating officer shall determine whether or not
there is sufficient ground to hold the respondent for trial.

Section 4. Resolution of investigating prosecutor and its review. — If the investigating prosecutor
finds cause to hold the respondent for trial, he shall prepare the resolution and information. He shall
certify under oath in the information that he, or as shown by the record, an authorized officer, has
personally examined the complainant and his witnesses; that there is reasonable ground to believe that a
crime has been committed and that the accused is probably guilty thereof; that the accused was informed
of the complaint and of the evidence submitted against him; and that he was given an opportunity to
submit controverting evidence. Otherwise, he shall recommend the dismissal of the complaint.

Within five (5) days from his resolution, he shall forward the record of the case to the provincial or city
prosecutor or chief state prosecutor, or to the Ombudsman or his deputy in cases of offenses cognizable
by the Sandiganbayan in the exercise of its original jurisdiction. They shall act on the resolution within ten
(10) days from their receipt thereof and shall immediately inform the parties of such action.
No complaint or information may be filed or dismissed by an investigating prosecutor without the prior
written authority or approval of the provincial or city prosecutor or chief state prosecutor or the
Ombudsman or his deputy.

Where the investigating prosecutor recommends the dismissal of the complaint but his recommendation
is disapproved by the provincial or city prosecutor or chief state prosecutor or the Ombudsman or his
deputy on the ground that a probable cause exists, the latter may, by himself, file the information against
the respondent, or direct any other assistant prosecutor or state prosecutor to do so without conducting
another preliminary investigation.

If upon petition by a proper party under such rules as the Department of Justice may prescribe or motu
proprio, the Secretary of Justice reverses or modifies the resolution of the provincial or city prosecutor or
chief state prosecutor, he shall direct the prosecutor concerned either to file the corresponding
information without conducting another preliminary investigation, or to dismiss or move for dismissal of
the complaint or information with notice to the parties. The same rule shall apply in preliminary
investigations conducted by the officers of the Office of the Ombudsman.

Section 5. Resolution of investigating judge and its review. — Within ten (10) days after the
preliminary investigation, the investigating judge shall transmit the resolution of the case to the provincial
or city prosecutor, or to the Ombudsman or his deputy in cases of offenses cognizable by the
Sandiganbayan in the exercise of its original jurisdiction, for appropriate action. The resolution shall state
the findings of facts and the law supporting his action, together with the record of the case which shall
include:
(a) the warrant, if the arrest is by virtue of a warrant;
(b) the affidavits, counter-affidavits and other supporting evidence of the parties;
(c) the undertaking or bail of the accused and the order for his release;
(d) the transcripts of the proceedings during the preliminary investigation; and
(e) the order of cancellation of his bail bond, if the resolution is for the dismissal of the complaint.

Within thirty (30) days from receipt of the records, the provincial or city prosecutor, or the Ombudsman or
his deputy, as the case may be, shall review the resolution of the investigating judge on the existence of
probable cause. Their ruling shall expressly and clearly state the facts and the law on which it is based
and the parties shall be furnished with copies thereof. They shall order the release of an accused who is
detained if no probable cause is found against him. (5a)

Section 8. Records. — (a) Records supporting the information or complaint. — An information or


complaint filed in court shall be supported by the affidavits and counter-affidavits of the parties and their
witnesses, together with the other supporting evidence and the resolution on the case.

(b) Record of preliminary investigation. — The record of the preliminary investigation, whether conducted
by a judge or a fiscal, shall not form part of the record of the case. However, the court, on its own initiative
or on motion of any party, may order the production of the record or any its part when necessary in the
resolution of the case or any incident therein, or when it is to be introduced as an evidence in the case by
the requesting party.

2. Conducted by Ombudsman/Special Prosecutor; RA 6770

Section 15. Powers, Functions and Duties. — The Office of the Ombudsman shall have the following
powers, functions and duties:

(1) Investigate and prosecute on its own or on complaint by any person, any act or omission of
any public officer or employee, office or agency, when such act or omission appears to be illegal,
unjust, improper or inefficient.t has primary jurisdiction over cases cognizable by the
Sandiganbayan and, in the exercise of this primary jurisdiction, it may take over, at any stage,
from any investigatory agency of Government, the investigation of such cases.

Administrative Order No. 07, Rule 11, Sections 1-7

Section 1. Grounds – A criminal complaint may be brought for an offense in violation of R.A. 3019, as
amended, R.A. 1379 as amended, R.A. 6713, Title VII, Chapter II, Section 2 of the Revised Penal Code,
and for such other offenses committed by public officers and employees in relation to office.

Section 2. Evaluation – Upon evaluating the complaint, the investigating officer shall recommend
whether it may be:
a) dismissed outright for want of palpable merit;
b) referred to respondent for comment;
c) indorsed to the proper government office or agency which has jurisdiction over the case;
d) forwarded to the appropriate office or official for fact-finding investigation;
e) referred for administrative adjudication; or f) subjected to a preliminary investigation.

Section 3. Preliminary investigation; who may conduct. Preliminary Investigation may be conducted
by any of the following:
1) Ombudsman Investigators;
2) Special Prosecuting Officers;
3) Deputized Prosecutors;
4) Investigating Officials authorized by law to conduct preliminary investigations; or
5) Lawyers in the government service, so designated by the Ombudsman.

Section 4. Procedure – The preliminary investigation of cases falling under the jurisdiction of the
Sandiganbayan and Regional Trial Courts shall be conducted in the manner prescribed in Section 3, Rule
112 of the Rules of Court, subject to the following provisions:

a) If the complaint is not under oath or is based only on official reports, the investigating officer shall
require the complainant or supporting witnesses to execute affidavits to substantiate the complaints.

b) After such affidavits have been secured, the investigating officer shall issue an order, attaching thereto
a copy of the affidavits and other supporting documents, directing the respondents to submit, within ten
(10) days from receipt thereof, his counter-affidavits and controverting evidence with proof of service
thereof on the complainant. The complainant may file reply affidavits within ten (10) days after service of
the counter affidavits.

c) If the respondents does not file a counter-affidavit, the investigating officer may consider the comment
filed by him, if any, as his answer to the complaint. In any event, the respondent shall have access to the
evidence on record.

d) No motion to dismiss shall be allowed except for lack of jurisdiction. Neither may a motion for a bill of
particulars be entertained. If respondents desires any matter in the complainant's affidavit to be clarified,
the particularization thereof may be done at the time of clarificatory questioning in the manner provided in
paragraph (f) of this section.

e) If the respondents cannot be served with the order mentioned in paragraph 6 hereof, or having been
served, does not comply therewith, the complaint shall be deemed submitted for resolution on the basis of
the evidence on the record.

f) If, after the filing of the requisite affidavits and their supporting evidences, there are facts material to the
case which the investigating officer may need to be clarified on, he may conduct a clarificatory hearing
during which the parties shall be afforded the opportunity to be present but without the right to examine or
cross-examine the witness being questioned. Where the appearance of the parties or witnesses is
impracticable, the clarificatory questioning may be conducted in writing, whereby the questions desired to
be asked by the investigating officer or a party shall be reduced into writing and served on the witness
concerned who shall be required to answer the same in writing and under oath.

g) Upon the termination of the preliminary investigation, the investigating officer shall forward the records
of the case together with his resolution to the designated authorities for their appropriate action thereon.
No information may be filed and no complaint may be dismissed without the written authority or approval
of the Ombudsman in cases falling within the jurisdiction of the Sandiganbayan, or of the proper Deputy
Ombudsman in all other cases.

Section 5. Cases falling under the jurisdiction of municipal trial courts. – Cases falling under the
jurisdiction of the Office of the Ombudsman which are cognizable by municipal trial courts, including those
subject to the Rule on Summary Procedure may only be filed in court by information approved by the
Ombudsman or the proper Deputy Ombudsman.

Section 6. Notice to parties. – The parties shall be served with a copy of the resolution as finally
approved by the Ombudsman or by the proper Deputy Ombudsman.

Section 7. Motion for reconsideration


a) Only one motion for reconsideration or reinvestigation of an approved order or resolution shall be
allowed, the same to be filled within five (5) days from notice thereof with the Office of the Ombudsman,
or the proper Deputy Ombudsman as the case may be, with corresponding leave of court in cases where
information has already been filed in court;

b) The filing of a motion for reconsideration/reinvestigation shall not bar the filing of the corresponding
information in Court on the basis of the finding of probable cause in the resolution subject of the motion.
(As amended by Administrative Order No. 15, dated February 16, 2000

Ombudsman and DOJ MOA

Under the MOA, the Ombudsman will have "primary jurisdiction in the conduct of preliminary investigation
and inquest proceedings over complaints for crimes cognizable by the Sandiganbayan."
Under Section 4 of Republic Act No. 8249, among the cases over which the Sandiganbayan has
exclusive jurisdiction are the following:

(I) Violations of R.A. 3019 (Anti-Graft and Corrupt Practices Act);


(II) R.A. 1379 (Forfeiture of Illegally Acquired Wealth);
(III) Crimes by public officers or employees embraced in Chapter II, Section 2, Title VII, Book II of the
Revised Penal Code (Crimes committed by Public Officers) namely:
(a) Direct Bribery,
(b) Indirect Bribery,
(c) Qualified Bribery,
(d) Corruption of Public Officials;
(IV) Other offenses or felonies whether simple or complexed with other crimes committed in relation to
their office by the public officials and employees mentioned above and cases involving government
officials and employees with Salary Grade 27 or higher for:
(a) Violation of R.A. 6713 (Code of Conduct and Ethical Standards);
(b) Violation of R.A. 7080 (The Plunder Law);
(c) Violation of R.A. 7659 (The Heinous Crime Law);
(d) R.A. 9160 (Violation of the Anti-Money Laundering Law when committed by a public officer);
(e) PD 46 (Prohibition Against Gift-Giving); and
(f) PD 749 (law granting immunity to bribe-givers who voluntarily give information).

It said that when complaints involving the above-mentioned cases are filed before the DOJ, the same
shall immediately be endorsed to the Ombudsman.

The two officials also agreed that the Ombudsman and the DOJ shall have concurrent jurisdiction over
cases involving public officials falling outside the exclusive jurisdiction of the Sandiganbayan.
However, the office where such complaint is filed for preliminary investigation shall acquire jurisdiction
over the complaint to the exclusion of the other.

Once the DOJ acquires jurisdiction over said cases, the Prosecutor General or provincial/city prosecutors
shall resolve them without need of approval from the Ombudsman. The Ombudsman shall conduct
reinvestigation, when ordered by the court, of cases which underwent preliminary investigation at the
Ombudsman but referred to the DOJ for prosecution.

The MOA also said that the Ombudsman shall ensure that a motion for reconsideration has been
resolved or the period for filing thereof has lapsed before endorsing an Information to the DOJ for filing
with the proper court.

 Procedure in cases not requiring preliminary investigation

A. MTC cases or those covered by Summary Procedure

Section 9. Cases not requiring a preliminary investigation nor covered by the Rule on Summary
Procedure. —
(a) If filed with the prosecutor. — If the complaint is filed directly with the prosecutor involving an offense
punishable by imprisonment of less four (4) years, two (2) months and one (1) day, the procedure
outlined in section 3(a) of this Rule shall be observed. The prosecutor shall act on the complaint based on
the affidavits and other supporting documents submitted by the complainant within ten (10) days from its
filing.

(b) If filed with the Municipal Trial Court. — If the complaint or information is filed directly with the
Municipal Trial Court or Municipal Circuit Trial Court for an offense covered by this section, the procedure
in section 3(a) of this Rule shall be observed. If within ten (10) days after the filing of the complaint or
information, the judge finds no probable cause after personally evaluating the evidence, or after
personally examining in writing and under oath the complainant and his witnesses in the form of
searching question and answers, he shall dismiss the same.

He may, however, require the submission of additional evidence, within ten (10) days from notice, to
determine further the existence of probable cause. If the judge still finds no probable cause despite the
additional evidence, he shall, within ten (10) days from its submission or expiration of said period, dismiss
the case. When he finds probable cause, he shall issue a warrant of arrest, or a commitment order if the
accused had already been arrested, and hold him for trial. However, if the judge is satisfied that there is
no necessity for placing the accused under custody, he may issue summons instead of a warrant of
arrest.

B. When person lawfully arrested without a warrant

1). Section 7. When accused lawfully arrested without warrant. — When a person is lawfully arrested
without a warrant involving an offense which requires a preliminary investigation, the complaint or
information may be filed by a prosecutor without need of such investigation provided an inquest has been
conducted in accordance with existing rules. In the absence or unavailability of an inquest prosecutor, the
complaint may be filed by the offended party or a peace office directly with the proper court on the basis
of the affidavit of the offended party or arresting officer or person.

2). DOJ Department Order No. 61, Sept. 21, 1993


Section 6 is called INQUEST PROCEEDINGS, related to Rule 113, Section 5 [a] and [b] on warrantless
arrest. Inquests proceedings follow in cases where persons are arrested without the benefit of an arrest
order or warrant, or are caught in the act of committing a criminal offense.

Only offenses that would require preliminary investigation will have to go through inquest. Those not
requiring preliminary investigation need not go through an inquest proceeding.

Here, there is no need for preliminary investigation because there is a deadline for the accused to be
detained. Otherwise the peace officer will be guilty of arbitrary detention – delay in the delivery.

The purpose of the inquest proceedings in these cases is that while the state acknowledges the law
enforcers’ authority to arrest and detain persons without a warrant, the state must also ensure that these
persons are not unlawfully detained, and that they are not denied due process. The inquest establishes
whether the evidence is sufficient enough to seek court approval to keep the person in detention.

Prosecutors have a heavy burden to oversee police investigations in cases involving inquest proceedings
(DOJ Circular 61 on New Rules on Inquest). Each police station or headquarters should in principle also
have designated inquest prosecutors to process inquest procedures with a schedule of assignments for
their regular inquest duties.

The inquest requires the prosecutors to resolve the complaint the police filed in a prescribed period,
which varies depending on the gravity of the offense. Cases punishable with light penalties must be
resolved in 12 hours; those punishable with correctional penalties within 18 hours; and those punishable
by afflictive or capital penalties, within 36 hours. If the inquest prosecutor fails to complete the
proceedings in the prescribed period then the person must be released

Case:

3). Leviste vs. Alameda

FACTS:
Jose Antonio C. Leviste (petitioner) was, by Information, charged with homicide for the death of Rafael de
las Alas on January 12, 2007 before the Regional Trial Court (RTC) of Makati City. Petitioner was placed
under police custody while confined at the Makati Medical Center. After petitioner posted a bond which
the trial court approved, he was released from detention, and his arraignment was set.
The private complainants-heirs of De las Alas filed, with the conformity of the public prosecutor, an Urgent
Omnibus Motion praying, inter alia, for the deferment of the proceedings to allow the public prosecutor to
re-examine the evidence on record or to conduct a reinvestigation to determine the proper offense.

ISSUE:
Whether or not in cases when an accused is arrested without a warrant, the remedy of preliminary
investigation belongs only to the accused.

RULING:
No. The Court holds that the private complainant can move for reinvestigation.
All criminal actions commenced by a complaint or information shall be prosecuted under the direction and
control of the public prosecutor The private complainant in a criminal case is merely a witness and not a
party to the case and cannot, by himself, ask for the reinvestigation of the case after the information had
been filed in court, the proper party for that being the public prosecutor who has the control of the
prosecution of the case. Thus, in cases where the private complainant is allowed to intervene by counsel
in the criminal action, and is granted the authority to prosecute, the private complainant, by counsel and
with the conformity of the public prosecutor, can file a motion for reinvestigation.

In such an instance, before a re-investigation of the case may be conducted by the public prosecutor, the
permission or consent of the court must be secured. If after such re-investigation the prosecution finds a
cogent basis to withdraw the information or otherwise cause the dismissal of the case, such proposed
course of action may be taken but shall likewise be addressed to the sound discretion of the court.
Once the trial court grants the prosecution’s motion for reinvestigation, the former is deemed to have
deferred to the authority of the prosecutorial arm of the Government. Having brought the case back to the
drawing board, the prosecution is thus equipped with discretion – wide and far reaching – regarding the
disposition thereof, subject to the trial court’s approval of the resulting proposed course of action.

D. Resolution of investigating prosecutor and its review

Section 4, Rule 112. Resolution of investigating prosecutor and its review. — If the investigating
prosecutor finds cause to hold the respondent for trial, he shall prepare the resolution and information. He
shall certify under oath in the information that he, or as shown by the record, an authorized officer, has
personally examined the complainant and his witnesses; that there is reasonable ground to believe that a
crime has been committed and that the accused is probably guilty thereof; that the accused was informed
of the complaint and of the evidence submitted against him; and that he was given an opportunity to
submit controverting evidence. Otherwise, he shall recommend the dismissal of the complaint.

Within five (5) days from his resolution, he shall forward the record of the case to the provincial or city
prosecutor or chief state prosecutor, or to the Ombudsman or his deputy in cases of offenses cognizable
by the Sandiganbayan in the exercise of its original jurisdiction. They shall act on the resolution within ten
(10) days from their receipt thereof and shall immediately inform the parties of such action.

No complaint or information may be filed or dismissed by an investigating prosecutor without the prior
written authority or approval of the provincial or city prosecutor or chief state prosecutor or the
Ombudsman or his deputy.

Where the investigating prosecutor recommends the dismissal of the complaint but his recommendation
is disapproved by the provincial or city prosecutor or chief state prosecutor or the Ombudsman or his
deputy on the ground that a probable cause exists, the latter may, by himself, file the information against
the respondent, or direct any other assistant prosecutor or state prosecutor to do so without conducting
another preliminary investigation.

If upon petition by a proper party under such rules as the Department of Justice may prescribe or motu
proprio, the Secretary of Justice reverses or modifies the resolution of the provincial or city prosecutor or
chief state prosecutor, he shall direct the prosecutor concerned either to file the corresponding
information without conducting another preliminary investigation, or to dismiss or move for dismissal of
the complaint or information with notice to the parties. The same rule shall apply in preliminary
investigations conducted by the officers of the Office of the Ombudsman.
E. Resolution of investigating judge and its review

Section 5, Rule 112. Resolution of investigating judge and its review. — Within ten (10) days after
the preliminary investigation, the investigating judge shall transmit the resolution of the case to the
provincial or city prosecutor, or to the Ombudsman or his deputy in cases of offenses cognizable by the
Sandiganbayan in the exercise of its original jurisdiction, for appropriate action. The resolution shall state
the findings of facts and the law supporting his action, together with the record of the case which shall
include:

(a) the warrant, if the arrest is by virtue of a warrant;


(b) the affidavits, counter-affidavits and other supporting evidence of the parties;
(c) the undertaking or bail of the accused and the order for his release;
(d) the transcripts of the proceedings during the preliminary investigation; and
(e) the order of cancellation of his bail bond, if the resolution is for the dismissal of the complaint.

Within thirty (30) days from receipt of the records, the provincial or city prosecutor, or the Ombudsman or
his deputy, as the case may be, shall review the resolution of the investigating judge on the existence of
probable cause. Their ruling shall expressly and clearly state the facts and the law on which it is based
and the parties shall be furnished with copies thereof. They shall order the release of an accused who is
detained if no probable cause is found against him.

F. When warrant of arrest may issue

Section 6, Rule 112. When warrant of arrest may issue. —


(a) By the Regional Trial Court. — Within ten (10) days from the filing of the complaint or information, the
judge shall personally evaluate the resolution of the prosecutor and its supporting evidence. He may
immediately dismiss the case if the evidence on record clearly fails to establish probable cause. If he
finds probable cause, he shall issue a warrant of arrest, or a commitment order if the accused has already
been arrested pursuant to a warrant issued by the judge who conducted the preliminary investigation or
when the complaint or information was filed pursuant to section 7 of this Rule. In case of doubt on the
existence of probable cause, the judge may order the prosecutor to present additional evidence within
five (5) days from notice and the issue must be resolved by the court within thirty (30) days from the filing
of the complaint of information.

(b) By the Municipal Trial Court. — When required pursuant to the second paragraph of section 1 of this
Rule, the preliminary investigation of cases falling under the original jurisdiction of the Metropolitan Trial
Court, Municipal Trial Court in Cities, Municipal Trial Court, or Municipal Circuit Trial Court may be
conducted by either the judge or the prosecutor. When conducted by the prosecutor, the procedure for
the issuance of a warrant or arrest by the judge shall be governed by paragraph (a) of this section. When
the investigation is conducted by the judge himself, he shall follow the procedure provided in section 3 of
this Rule. If the findings and recommendations are affirmed by the provincial or city prosecutor, or by the
Ombudsman or his deputy, and the corresponding information is filed, he shall issue a warrant of arrest.
However, without waiting for the conclusion of the investigation, the judge may issue a warrant of arrest if
he finds after an examination in writing and under oath of the complainant and his witnesses in the form
of searching question and answers, that a probable cause exists and that there is a necessity of placing
the respondent under immediate custody in order not to frustrate the ends of justice.

(c) When warrant of arrest not necessary. — A warrant of arrest shall not issue if the accused is already
under detention pursuant to a warrant issued by the municipal trial court in accordance with paragraph (b)
of this section, or if the complaint or information was filed pursuant to section 7 of this Rule or is for an
offense penalized by fine only. The court shall then proceed in the exercise of its original jurisdiction.
G. When accused lawfully arrested without a warrant

Section 7, Rule 112. When accused lawfully arrested without warrant. — When a person is lawfully
arrested without a warrant involving an offense which requires a preliminary investigation, the complaint
or information may be filed by a prosecutor without need of such investigation provided an inquest has
been conducted in accordance with existing rules. In the absence or unavailability of an inquest
prosecutor, the complaint may be filed by the offended party or a peace office directly with the proper
court on the basis of the affidavit of the offended party or arresting officer or person.
Before the complaint or information is filed, the person arrested may ask for a preliminary investigation in
accordance with this Rule, but he must sign a waiver of the provisions of Article 125 of the Revised Penal
Code, as amended, in the presence of his counsel. Notwithstanding the waiver, he may apply for bail and
the investigation must be terminated within fifteen (15) days from its inception.
After the filing of the complaint or information in court without a preliminary investigation, the accused
may, within five (5) days from the time he learns of its filing, ask for a preliminary investigation with the
same right to adduce evidence in his defense as provided in this Rule. (7a; sec. 2, R.A. No. 7438).

H. Records

Section 8, Rule 112. Records. —


(a) Records supporting the information or complaint. — An information or complaint filed in court shall be
supported by the affidavits and counter-affidavits of the parties and their witnesses, together with the
other supporting evidence and the resolution on the case.

(b) Record of preliminary investigation. — The record of the preliminary investigation, whether conducted
by a judge or a fiscal, shall not form part of the record of the case. However, the court, on its own initiative
or on motion of any party, may order the production of the record or any its part when necessary in the
resolution of the case or any incident therein, or when it is to be introduced as an evidence in the case by
the requesting party.

I. Cases not requiring a preliminary investigation

Section 9, Rule 112. Cases not requiring a preliminary investigation nor covered by the Rule on
Summary Procedure. —
(a) If filed with the prosecutor. — If the complaint is filed directly with the prosecutor involving an offense
punishable by imprisonment of less four (4) years, two (2) months and one (1) day, the procedure
outlined in section 3(a) of this Rule shall be observed. The prosecutor shall act on the complaint based on
the affidavits and other supporting documents submitted by the complainant within ten (10) days from its
filing.

(b) If filed with the Municipal Trial Court. — If the complaint or information is filed directly with the
Municipal Trial Court or Municipal Circuit Trial Court for an offense covered by this section, the procedure
in section 3(a) of this Rule shall be observed. If within ten (10) days after the filing of the complaint or
information, the judge finds no probable cause after personally evaluating the evidence, or after
personally examining in writing and under oath the complainant and his witnesses in the form of
searching question and answers, he shall dismiss the same.

He may, however, require the submission of additional evidence, within ten (10) days from notice, to
determine further the existence of probable cause. If the judge still finds no probable cause despite the
additional evidence, he shall, within ten (10) days from its submission or expiration of said period, dismiss
the case. When he finds probable cause, he shall issue a warrant of arrest, or a commitment order if the
accused had already been arrested, and hold him for trial. However, if the judge is satisfied that there is
no necessity for placing the accused under custody, he may issue summons instead of a warrant of
arrest.

J. Remedies from preliminary investigation

A. Appeal

Cases:

1. Dimatulac vs Villon

FACTS:
In the prosecution of the Yabuts for the murder of Dimatulac, the Office of the Public Prosecutor
(particularly the Asst Prosecutor) and two Judges (who handled the case) committed serious procedural
flaws resulting in the impairment of due process (prejudicial to both the offended party and the accused).
Procedural irregularities in the Office of the Provincial Prosecutor:

Warrants of arrest were issued by the MCTC, with no bail recommended, but the Yabuts were not
arrested or were never brought unto the custody of the law. Yet, Asst Fiscal Alfonso-Reyes conducted a
reinvestigation. Though a prosecutor may disagree with the findings of the judge who conducted the
preliminary investigation (and conduct his own), the circumstance that the accused waived the filing of
their counter-affidavits left Alfonso-Reyes no other choice but to sustain the MCTC findings—which she
did not do. And later on, Alfonso-Reyes allowed the Yabuts to file their counter-affidavits without first
demanding that they surrender by virtue of the standing warrants of arrest.

Alfonso-Reyes recommended a bond of 20k for the Yabuts despite the fact that they were charged of
homicide and that they were fugitives from justice (having avoided service of warrant of arrest).
Alfonso-Reyes was aware of the private prosecution’s appeal to the DOJ from her resolution. (The
subsequent resolution of the DOJ Secretary exposed her blatant errors.) And despite the pending appeal,
she filed the Information. It would be more prudent to wait for the DOJ resolution.
Office of the Prosecutor did not even inform the trial court of the pending appeal to the DOJ Secretary.
Judge Roura’s procedural lapses:

 Deferred resolution on the motion for a hold departure order until “such time that all the accused
who are out on bail are arraigned
 Denied the motion to defer proceedings for the reason that private prosecution has not shown
any indication that the appeal was given due course by DOJ.

Judge Villon’s procedural lapses:

 Ordered arraignment despite: a motion to defer proceedings; a ten-day period with which the
complainants can file petition with the CA; resolution of the CA ordering the Yabuts to comment
on the complainants’ action; pending appeal with the DOJ.

ISSUE:
Can the orders of Judge Roura and Judge Villon be sustained despite procedural defects?

RULING:
No. The orders of Judge Roura denying Motion to Defer proceedings are void and set aside. The order of
Judge Villon on the arraignment, and the subsequent arraignment of the Yabuts are void and set aside.
Office of the Provincial Prosecutor is ordered to comply with the DOJ Secretary’s resolution.

Prosecutors are the representatives not of an ordinary party to a controversy, but of a sovereignty whose
obligation to govern impartially is as compelling as its obligation to govern at all; and whose interest in a
criminal prosecution is not that it shall win every case but that justice be done. They are servants of the
law whose two-fold aim is that guilt shall not escape and innocence shall not suffer.
The judge should always be imbued with a high sense of duty and responsibility in the discharge of his
obligation to promptly and properly administer justice. The judge’s action must not impair the substantial
rights of the accused, nor the right of the State and offended party.

When the State is deprived of due process in a criminal case by reason of grave abuse of discretion on
the part of the trial court, the acquittal of the accused or dismissal of the case is void.

2. Ty vs. NBI

FACTS:
Petitioners are stockholders of Omni Gas Corporation ("Omni"). They are being suspected of engaging in
illegal trading of petroleum products and underfilling of branded LPG cylinders in violation of B.P. 33, as
amended by P.D. 1865. NBI Agents Marvin De Jemil and Edgardo Kawada conducted surveillance
operations on Omni. On 15 April 2004, the NBI Agents carried out a test-buy. Using eight branded LPG
cylinders from Shell, Petron and Total, they went to Omni for refilling. Omni refilled the cylinders. The NBI
agents paid more than P1500. LPG Inspector Noel Navio found that the LPG cylinders were without LPG
valve seals and one of the cylinders was actually underfilled.

On 28 April 2004, Agent De Jemil obtained a search warrant from Pasig RTC branch 167. The NBI seized
several items from Omni's premises. Subsequently, Agent De Jemil filed his Complaint-Affidavit before
the DOJ. The Assistant City Prosecutor of Pasig found probable cause for violation of BP 33. This was
later approved by Chief State Prosecutor Jovencito Zuno.

Petitioners appealed the decision to the Secretary of Justice, who later reversed the decision of the Office
of the Chief State Prosecutor. NBI Agent De Jemil moved for reconsideration. Denied. He thus filed a
petition for certiorari under Rule 65 with the Court of Appeals.

The Court of Appeals affirmed the decision of Secretary of Justice. It later reversed itself and reinstated
the Resolution of the Chief State Prosecutor.

ISSUE:
Whether or not the petition for certiorari with the Court of Appeals was proper even if Agent De Jemil did
not appeal to the Office of the President.

RULING:
YES. The determination of probable cause by the public prosecutor, and, later on, by the Secretary of
Justice, is subject to judicial review where it is established that grave abuse of discretion tainted the
determination. The aggrieved party need not resort to the Office of the President before availing of judicial
remedies because the Secretary of Justice is an alter ego of the President who may opt to exercise or not
to exercise his or her power of review over the former’s determination in criminal investigation cases.
Also, under the doctrine of qualified political agency, the determination of probable cause by the
Secretary of Justice is presumably that of the Chief Executive unless disapproved or reprobated by the
latter.

B. Reinvestigation/Preliminary Investigation

1. Crespo vs. Mogul

FACTS:
Petitioner Mario Crespo was accused for Estafa in the Circuit Criminal Court of Lucena City. When the
case was set for arraignment, the accused filed a motion for defer arraignment on the ground that there
was a pending petition for review filed with the Secretary of Justice. However, Justice Mogul denied the
motion, but the arraignment was deferred in a much later date to afford time for the petitioner to elevate
the matter to the appellate court.
The accused filed a petition for certiorari and prohibition with prayer for a preliminary writ of injunction to
the CA. The CA ordered the trial court to refrain from proceeding with the arraignment until further orders
of the Court. Undersecretary of Justice, Hon. Catalino Macaraig Jr., resolved the petition for review
reversed the resolution of the office of the Provincial Fiscal and directed the Fiscal to move for immediate
dismissal of the information filed against the accused. Judge Mogul denied the motion for dismissal of
the case ad set the arraignment. The accused then filed a petition for Certiorari, prohibition and
mandamus with petition for the issuance of preliminary writ of prohibition and/or temporary restraining
order in the CA. The CA dismissed the order and lifted the restraining order.

ISSUE:
Whether the trial court may refuse to grant a motion to dismiss filed by the Fiscal under orders from the
Secretary of Justice and insists on arraignment and trial on the merits.

RULING:
It is a cardinal principle that all criminal actions either commenced by complaint or by information shall be
prosecuted under the direction and control of the fiscal. The institution of a criminal action depends upon
the sound discretion of the fiscal. The reason for placing the criminal prosecution under the direction and
control of the fiscal is to prevent malicious or unfounded prosecution by private persons. 19 It cannot be
controlled by the complainant.

However, the action of the fiscal or prosecutor is not without any limitation or control. The same is subject
to the approval of the provincial or city fiscal or the chief state prosecutor as the case maybe and it may
be elevated for review to the Secretary of Justice who has the power to affirm, modify or reverse the
action or opinion of the fiscal. Consequently the Secretary of Justice may direct that a motion to dismiss
the case be filed in Court or otherwise, that an information be filed in Court.

The filing of a complaint or information in Court initiates a criminal action. The Court thereby acquires
jurisdiction over the case, which is the authority to hear and determine the case. The preliminary
investigation conducted by the fiscal for the purpose of determining whether a prima facie case exists
warranting the prosecution of the accused is terminated upon the filing of the information in the proper
court.

2. Roberts vs. CA

FACTS:
Petitioners, who are corporate officers and members of the Board of Pepsi Cola Products Phils., Inc. were
prosecuted in connection with the Pepsi “Number Fever” promotion by handlers of the supposedly
winning “349” Pepsi crowns. Of the four cases filed against the petitioners, probable cause was found by
the investigating prosecutor only for the crime of estafa, but not for the other alleged offenses.

The information was filed with the trial court without anything accompanying it. A copy of the investigating
prosecutor’s Joint Resolution was forwarded to and received by the trial court. However, no affidavits of
the witnesses, transcripts of stenographic notes of the proceedings during the preliminary investigation, or
other documents submitted in the course thereof were found in the records of the case.

Petitioners Roberts, et al. filed a petition for review to the Department of Justice seeking the reversal of
the finding of probable cause by the investigating prosecutor. They also moved for the suspension of the
proceedings and the holding in abeyance of the issuance of warrants of arrest against them. Meanwhile,
the public prosecutor also moved to defer the arraignment of the accused-appellants pending the final
disposition of the appeal to the Department of Justice.

Respondent Judge Asuncion issued the challenged order denying, on the basis of Crespovs. Mogul, the
foregoing motions respectively filed by the petitioners and the public prosecutor, and directing the
issuance of the warrants of arrest and setting the arraignment. In part, respondent judge stated in his
order that since the case is already pending in this Court for trial, following whatever opinion the
Secretary of Justice may have on the matter would undermine the independence and integrity of the
court. To justify his order, he quoted the ruling of the Supreme Court in Crespo, which stated:

"In order therefor to avoid such a situation whereby the opinion of the Secretary of
Justice who reviewed the action of the fiscal may be disregarded by the trial court, the Secretary
of Justice should, as far as practicable, refrain from entertaining a petition for review or appeal
from the action of the fiscal, when the complaint or information has already been filed in Court.
The matter should be left entirely for the determination of the Court.

ISSUE:
Whether or not the judge abused his discretion in denying the motion to suspend proceedings.
.
RULING:
YES, Judge Asuncion committed grave abuse of discretion in denying, on the basis of Crespo vs.
Mogul, the motions to suspend proceedings and hold in abeyance the issuance of warrants of arrest and
to defer arraignment until after the petition for review filed with the DOJ shall have been resolved.
There is nothing in Crespo vs. Mogul which bars the DOJ from taking cognizance of an appeal, by way of
a petition for review, by an accused in a criminal case from an unfavorable ruling of the investigating
prosecutor. It merely advised the DOJ to, as far as practicable, refrain from entertaining a petition for
review or appeal from the action of the fiscal, when the complaint or information has already been filed in
Court.

Whether the DOJ would affirm or reverse the challenged Joint Resolution is still a matter of guesswork.
Accordingly, it was premature for respondent Judge Asuncion to deny the motions to suspend
proceedings and to defer arraignment.

The real and ultimate test of the independence and integrity of this court is not the filing of the
aforementioned motions to suspend proceedings and issuance of warrants of arrest and to defer
arraignment] at that stage but the filing of a motion to dismiss or to withdraw the information on the basis
of a resolution of the petition for review reversing the Joint Resolution of the investigating prosecutor.
However, once a motion to dismiss or withdraw the information is filed the trial judge may grant or deny it,
not out of subservience to the Secretary of Justice, but in faithful exercise of judicial prerogative.

3. Dungog vs. CA

FACTS:
On October 9, 1985, First Assistant Provincial Fiscal of Bohol Angel S. Ucat, Jr. issued a resolution
finding a prima facie case for the filing of an information for estafa against Pantaleon del Rosario. The
same resolution was approved by the respondent Provincial Fiscal of Bohol, Enrique B. Inting.

An information charging the private respondent with estafa was then filed with the Regional Trial Court of
Bohol.

The private respondent then filed a Motion For Reinvestigation with the public respondent Provincial
Fiscal. The same was granted and this time, the respondent Provincial Fiscal found no prima facie case
against the same private respondent. An Omnibus Motion For Postponement Of Arraignment And To
Allow Withdrawal Of Information was then filed.

Presiding Judge of the Regional Trial Court of Bohol resolved to deny the respondent Provincial Fiscal's
Motion to Withdraw Information as well as the Motion for Reconsideration.

ISSUE:
Whether or not the Presiding Judge of the RTC of Bohol committed grave abuse of discretion in denying
the motion for postponement of arraignment and withdrawal of information.

RULING:
No, the rule is now well-settled that once a complaint or information is filed in court any disposition of the
case as to its dismissal or the conviction or acquittal of the accused rests in the sound discretion of the
court. Although the fiscal retains the direction and control of the prosecution of criminal cases even while
the case is already in court he cannot impose his opinion on the trial court. For while it is true that the
fiscal has the quasi-judicial discretion to determine whether or not a criminal case should be filed in court,
once the case had already been brought to court, whatever disposition the fiscal may deem proper
thereafter should be addressed to the court for its consideration and approval.

4. Velasquez vs. Undersecretary of Justice

FACTS:
Felix A. Velasquez, as Executive Vice-President/Managing Director of Techtrade, filed a complaint for
estafa against Avila in the Manila City Fiscal's Office. Assistant Fiscal Romulo Lopez dismissed the
complaint. However, upon review by the Chief, Investigation Division of the City Fiscal's Office, the latter
set aside Fiscal Lopez' resolution and ordered the filing of an information for estafa against Avila in the
RTC.

Before arraignment, Avila filed in the DOJ a petition for review which Velasquez opposed.

Justice Undersecretary Silvestre Bello III denied the petition for review.

A motion for reconsideration of the denial did not prosper.

Avila filed a second motion for reconsideration which the Undersecretary of Justice, Honorable Artemio
Tuquero granted on January 4, 1989. He directed the City Fiscal to conduct a reinvestigation of this case
to afford respondent to properly present evidence that he was duly authorized to pay the subject creditors
and for complainant to rebut the same with controverting evidence, and thereafter to resolve the case
anew on the basis of all the evidence adduced. The complainant filed a motion for reconsideration of that
resolution but it was denied. Hence, this petition for certiorari.

ISSUE:
WON the Undersecretary of Justice has the authority in ordering the re-investigation of the criminal case.

RULING:
No, the Undersecretary of Justice gravely abused his power in ordering the same. In the case of Crespo
vs. Mogul, where the SC ruled that once the information is filed in court, the court acquires complete
jurisdiction over it. A motion for reinvestigation should, after the court had acquired jurisdiction over the
case, be addressed to the trial judge and to him alone. Neither the Secretary of Justice, the State
Prosecutor, nor the Fiscal may interfere with the judge's disposition of the case, much less impose upon
the court their opinion regarding the guilt or innocence of the accused, for the court is the sole judge of
that. In order therefore to avoid such a situation whereby the opinion of the Secretary of Justice who
reviewed the action of the fiscal may be disregarded by the trial court, the Secretary of Justice should, as
far as practicable, refrain from entertaining a petition for review or appeal from the action of the fiscal,
when the complaint or information has already been filed in Court. The matter should be left entirely for
the determination of the Court.

Thus, the Undersecretary of Justice gravely abused his discretion in ordering the re-investigation of the
criminal case against Avila after it had been filed in court.

5. People vs. Beriales

FACTS:
Ricardo Beriales, Benedicto Custodio, and Pablito Custodio herein appellants were charged with the
crime of murder in an information filed by the City Fiscal of Ormoc City.
Appellants' counsel moved for a reinvestigation of said case, along with two other related cases which the
court a quo granted in its Order.
The trial court postponed the hearings of the case in view of the City Fiscal's motion for a deferment of
the hearing or trial. However, the trial court, motu proprio cancelled the aforesaid hearings and instead
reset the arraignment and trial of the case.

At the hearing, appellants' counsel manifested to the court that pursuant to its approval of his motion for
reinvestigation, the City Fiscal had set the reinvestigation and had already issued the corresponding
subpoena to secure the attendance of the witnesses. The trial court, however, re-scheduled the hearing,
relying on the mandate of the New Constitution that "all persons shall have the right to a speedy
disposition of their cases before all judicial, quasi-judicial, or administrative bodies".

Thereafter, the court proceeded with the arraignment and trial in the absence of the Fiscal and its report
on re-investigation, and over the disagreement of the defense. The CFI of Leyte relied on the private
prosecutor being authorized by the Fiscal to present evidence and the defense presumed to have waived
its right over its disagreement. The Trial then proceeded and the court found Ricardo Beriales, Benedicto
Custodio, and Pablito Custodio guilty of the offense.

ISSUE:
Whether or not there was a grave abuse of discretion of the trial court in proceeding with the trial despite
the absence of the Fiscal and its report on re-investigation.

RULING:
Yes. After the trial court granted the appellants' motion for reinvestigation, it became incumbent upon the
court to hold in abeyance the arraignment and trial of the case until the City Fiscal shall have conducted
and made his report on the result of such reinvestigation. That was a matter of duty on its part, not only to
be consistent with its own order but also to do justice aid at the same time to avoid a possible miscarriage
of justice. It should be borne in mind that it was entirely possible for the City Fiscal to modify or change
his conclusion after conducting the reinvestigation.

Therefore, when the trial court ignored the appellants' manifestations objecting to the arraignment and the
trial of the case, until after the City Fiscal shall have rendered a resolution on his reinvestigation, but
instead considered such manifestations on their part as a plea of riot guilty and proceeded to try the case,
received the evidence for the prosecution, and then rendered judgment against them on the basis thereof,
it committed a serious irregularity which nullifies the proceedings below because such a procedure is
repugnant to the due process clause of the Constitution.

 Effect of absence of or irregularity of preliminary investigation:

Cases:

1. Go vs. CA

FACTS:
Rolito Go while traveling in the wrong direction on a one-way street, nearly bumped Eldon Maguan’s car.
Go alighted from his car, shot Maguan and left the scene. A security guard at a nearby restaurant was
able to take down petitioner’s car plate number. The police arrived shortly thereafter at the scene of the
shooting and a manhunt ensued.

Six days after, petitioner presented himself before the San Juan Police Station to verify news reports that
he was being hunted by the police; he was accompanied by two lawyers. The police forthwith detained
him. An eyewitness to the shooting, who was at the police station at that time, positively identified
petitioner as the gunman.
Petitioner then posted bail, and the prosecutor filed the case to the lower court, setting and commencing
trial without preliminary investigation. Prosecutor reasons that the petitioner has waived his right to
preliminary investigation as bail has been posted and that such situation, that petitioner has been
arrested without a warrant lawfully, falls under Section 5, Rule 113 and Section 7, Rule 112 of The 1985
Rules of Criminal Procedure which provides for the rules and procedure pertaining to situations of lawful
warrantless arrests.

Petitioner argues that he was not lawfully arrested without warrant because he went to the police station
six days after the shooting which he had allegedly perpetrated. Thus, petitioner argues, the crime had not
been “just committed” at the time that he was arrested. Moreover, none of the police officers who arrested
him had been an eyewitness to the shooting of Maguan and accordingly none had the “personal
knowledge” required for the lawfulness of a warrantless arrest. Since there had been no lawful
warrantless arrest, Section 7, Rule 112 of the Rules of Court which establishes the only exception to the
right to preliminary investigation, could not apply in respect of petitioner.

ISSUE:
Whether petitioner had effectively waived his right to preliminary investigation when he posted bail.

RULING:
No. In the circumstances of this case, the Court does not believe that by posting bail, petitioner had
waived his right to preliminary investigation. In People v. Selfaison, the Court held that appellants there
had waived their right to preliminary investigation because immediately after their arrest, they filed bail
and proceeded to trial without previously claiming that they did not have the benefit of a preliminary
investigation.

In the instant case, petitioner Go asked for release on recognizance or on bail and for preliminary
investigation in one omnibus motion. He had thus claimed his right to preliminary
investigation before respondent Judge approved the cash bond posted by petitioner and ordered his
release. Accordingly, the Court cannot reasonably imply waiver of preliminary investigation on the part of
petitioner. In fact, when the Prosecutor filed a motion in court asking for leave to conduct preliminary
investigation, he clearly if impliedly recognized that petitioner’s claim to preliminary investigation was a
legitimate one.

2. Socrates vs. Sandiganbayan

FACTS:
Petitioner, who is the incumbent governor of Palawan, was first elected governor of the said province in
1968 and was again re-elected in both the 1971 and 1980 elections, until he was replaced by private
complainant Victoriano Rodriguez as Officer-in-Charge Governor after the EDSA revolution in February
1986. Subsequently, both petitioner and Rodriguez ran for governor in the 1988 elections where the latter
emerged victorious.

In the 1992 synchronized national and local elections, the two again contested the gubernatorial post and
this time, it was petitioner who won. At the time Rodriguez was still the OIC Governor of the province, the
Provincial Government of Palawan, as represented by Rodriguez and the Provincial Board members of
Palawan, filed before the Office of the Tanodbayan two complaints. The first complaint charged petitioner
with violation of section 3(b) of RA no. 3019, otherwise known as the Anti-Graft and Corrupt Practices
Act, and the second charged petitioner, together with several other provincial officers, with violation of
section 3(a) and (g) of the same law. On December 23, 1994, respondent court, without ruling on
petitioners motion to include co-principals, issued its questioned resolution granting the motion to
suspend pendente lite and ordering the suspension of petitioner as Provincial Governor of Palawan for a
period of 90 days from notice.

ISSUE:
Whether or not the absence of a preliminary investigation in an offense that requires a preliminary
investigation impair the validity of the information or render it defective.
RULING:
No. It has been consistently held that the absence of a preliminary investigation does not impair the
validity of the criminal information or render it defective. Neither will the absence of a preliminary
investigation, assuming that it is necessary to conduct a new one, affect the validity of the information
filed against petitioner. Dismissal of the case is not the remedy. It is not a ground for the quashal of a
complaint or information. The proper course of action that should be taken is for the Sandiganbayan to
hold in abeyance the proceedings upon such information and to remand the case to the office of the
Ombudsman for him or the Special Prosecutor to conduct a preliminary investigation, if the accused
actually makes out a case justifying such relief.

C. Reconsideration

D. Injuction in proper cases

1. Primicias vs. Pangasinan

FATCS:
Juan Augusta B. Primacias, plaintiff-appellee, was driving his car within the jurisdiction of Urdaneta when
a member of Urdaneta's Municipal Police asked him to stop. He was told, upon stopping, that he had
violated Municipal Ordinance No. 3, Series of 1964, and more particularly, for overtaking a truck. The
policeman then asked for plaintiff's license which he surrendered, and a temporary operator's permit was
issued to him.

Thereafter, a criminal complaint was filed in the Municipal Court against Primicias.
Due to the institution of the criminal case, plaintiff Primicias initiated an action for the annulment of said
ordinance with prayer for the issuance of preliminary injunction for the purpose of restraining defendants
Municipality of Urdaneta, Mayor Perez, Police Chief Suyat, Judge Soriano and Patrolman Andrada from
enforcing the ordinance. The writ was issued and Judge Soriano was enjoined from further proceeding in
the criminal case.

After trial, the Court of First Instance rendered the questioned decision holding that the ordinance was
null and void and had been repealed by Republic Act No. 4136, otherwise known as the Land
Transportation and Traffic Code.

Appellants contend that the Ordinance is valid, being "patterned after and based on Section 53, par. 4 of
Act No. 3992, as amended (Revised Motor Vehicle Law)." In so arguing, appellants fail to note that Act
No. 3992 has been superseded by Republic Act No. 4136, the Land Transportation and Traffic Code,
which became effective on June 20, 1964, about three months after the questioned ordinance was
approved by Urdaneta's Municipal Council. The explicit repeal of the aforesaid Act is embodied in Section
63, Republic Act No. 4136, to wit:

Act No. 3992 as amended, and all laws, executive orders, ordinance, resolutions, regulations or paints
thereof in conflict with the provisions of this Act are repealed.
An essential requisite for a valid ordinance is, among others, that it must not contravene the statute, for it
is a fundamental principle that municipal ordinances are inferior in status and subordinate to the laws of
the state. Following this general rule, whenever there is a conflict between an ordinance and a statute,
the ordinance must give way
.
ISSUE:
Whether or not a Writ of Injunction may restrain proceedings in Criminal Case.
RULING:
On the issue of whether a writ of injunction can restrain the proceedings in Criminal Case No. 3140, the
general rule is that ordinarily, criminal prosecution may not be blocked by court prohibition or injunction.

Exceptions however are allowed in the following instances:


1. for the orderly administration of justice;
2. to prevent the use of the strong arm of the law in an oppressive and vindictive manner;
3. to avoid multiplicity of actions;
4. to afford adequate protection to constitutional rights;
5. in proper cases, because the statute relied upon is unconstitutional or was held invalid.

The local statute or ordinance at bar being invalid, the exception just cited obtains in this case. Hence, the
lower court did not err in issuing the writ of injunction against defendants.

Moreover, considering that our law on municipal corporations is in principle patterned after that of the
United States, it would not be amiss for the court to adopt in this instance the ruling that to enjoin the
enforcement of a void ordinance, injunction has frequently been sustained in order to prevent a
multiplicity of prosecutions under it.

2. Guingona, Jr. vs. City Fiscal

FACTS:
David invested several deposits with the Nation Savings and Loan Association [NSLA]. He said that he
was induced into making said investments by an Australian national who was a close associate of the
petitioners [NSLA officials]. On March 1981, NSLA was placed under receivership by the Central Bank, so
David filed claims for his and his sister’s investments.

Guingona and Martin, upon David’s request, assumed the bank’s obligation to David by executing a joint
promissory note.

David received a report that only a portion of his investments was entered in the NSLA records.
David then filed I.S. No. 81-31938 in the Office of the City Fiscal, which case was assigned to Asst. City
Fiscal Lota for preliminary investigation. David charged petitioners with estafa and violation of Central
Bank Circular No. 364 and related regulations on foreign exchange transactions.

Petitioners moved to dismiss the charges against them for lack of jurisdiction because David's claims
allegedly comprised a purely civil obligation, but the motion was denied. After the presentation of David's
principal witness, petitioners filed this petition for prohibition and injunction.

A TRO was issued ordering the respondents to refrain from proceeding with the preliminary investigation
in I.S. No. 81-31938.

ISSUE:
Whether or not the TRO was correctly issued.

RULING:
Yes. Although as a general rule, criminal prosecution may not be blocked by court prohibition or
injunction, there are recognized exceptions to the abovesaid rule:
1. For the orderly administration of justice
2. To prevent the use of the strong arm of the law in an oppressive and vindictive manner
3. To avoid multiplicity of actions
4. To afford adequate protection to constitutional rights
5. In proper cases, because the statute relied upon is unconstitutional or was held invalid.
When David invested his money on time and savings deposits with NSLA, the contract that was perfected
was a contract of simple loan or mutuum and not a contract of deposit. The relationship between David
and NSLA is that of creditor and debtor. While the Bank has the obligation to return the amount
deposited, it has no obligation to return or deliver the same money that was deposited.

NSLA’s failure to return the amount deposited will not constitute estafa through misappropriation, but it
will only give rise to civil liability over which the public respondents have no jurisdiction.
Considering that petitioners’ liability is purely civil in nature and that there is no clear showing that they
engaged in foreign exchange transactions, public respondents acted without jurisdiction when they
investigated the charges against the petitioners.

Thus, public respondents should be restrained from further proceeding with the criminal case for to allow
the case to continue would work great injustice to petitioners and would render meaningless the proper
administration of justice.

3. People vs. Grey

FACTS:
An Information for Murder was filed against respondent Joseph Grey, former Mayor of San Jorge, Samar;
his son, respondent Francis Grey; and two others for the death of Rolando Diocton before the RTC of
Gandara, Samar. The Information was accompanied by other supporting documents and a motion for the
issuance of a warrant of arrest.

Meanwhile, Presiding Judge Rosario Bandal denied the motion for the issuance of a warrant of arrest.
Judge Bandal found the prosecution’s evidence to be insufficient to link respondents to the crime
charged. She directed the prosecution to present, within five days, additional evidence. Later, the judge
inhibited.

Thereafter, the venue was changed and Judge Naviadad continued the proceedings of the case.
Respondents filed a petition for certiorari seeking TRO and preliminary injunction alleging that the filing of
the murder charge are based on perjured statements since Joseph Grey announced his candidacy for the
Congressional election.

The CA held that Judge Naviadad failed to abide with the constitutional mandate of personally examining
the existence of probable cause. Thus, this petition.

ISSUE:
Whether or not Judge Naviadad gravely abuse his discretion for issuing the warrant of arrest and for
seeking a TRO.

RULING:
No. What the Constitution underscores is the exclusive and personal responsibility of the issuing judge to
satisfy himself of the existence of probable cause. In satisfying himself of the existence of probable cause
for the issuance of a warrant of arrest, the judge is not required to personally examine the complainant
and his witnesses. Following established doctrine and procedure, he shall:
(1) personally evaluate the report and the supporting documents submitted by the fiscal regarding the
existence of probable cause and, on the basis thereof, issue a warrant of arrest; or
(2) if on the basis thereof he finds no probable cause, he may disregard the fiscal’s report and require the
submission of supporting affidavits of witnesses to aid him in arriving at a conclusion as to the existence
of probable cause.

He should not rely solely on the report of the investigating prosecutor. It is not mandatory in the
determination of probable cause for the issuance of the warrant of arrest.
In this case, the judge, upon his personal examination of the complaint and evidence before him,
determined that there was probable cause to issue the warrants of arrest after the provincial prosecution,
based on the affidavits presented by complainant and her witnesses, found probable cause to file the
criminal Information. This finding of the Provincial Prosecutor was affirmed by the Secretary of Justice.

4. Brocka vs. Enrile

FACTS:
Jeepney was strike called by the Alliance of Concerned Transport Organization (ACTO), a demonstration
held in sympathy of this strike, forcibly and violently dispersed a petitioners arrested by Northern Police
District Officers.

Petitioners were charged with Illegal Assembly before RTC, but were released on bail except for Lino
Brocka, Ben Cervantes, Cosme Garcia and Rodolfo Santos (Brocka, et al.), who were charged as leaders
of the offense of Illegal Assembly for whom no bail was recommended.

Urgent petition for bail was filed before the RTC. The RTC Judge Miriam Defensor Santiago ordered
Brocka, et al’s provisional release. Thereafter Brocka, et al filed respective bail bonds.
However, despite service of release order, Brocka, et al remained in detention. Respondents-police
officers invoked Preventive Detention Action (PDA) allegedly issued against Brocka, et al on Jan.
28, 1985. Neither original nor certified true copy of this PDA was shown to Brocka, et al.

Furthermore, Brocka, et al. were subsequently charged on February 11, 1985 with Inciting to Sedition,
without prior notice to their counsel. The original informations filed recommended no bail. The
circumstances surrounding the hasty filing of this second offense are cited by Brocka, et al.

They were released provisionally only on February 14, 1985, on orders of then President Ferdinand E.
Marcos, hence, this petition.

Brocka, et al. contend that respondents' manifest bad faith and/or harassment are sufficient bases for
enjoining their criminal prosecution, aside from the fact that the second offense of inciting to sedition is
illegal, since it is premised on one and the same act of attending and participating in the
ACTO jeepney strike. They maintain that while there may be a complex crime from a single act (Art. 48,
RTC), the law does not allow the splitting of a single act into two offenses and filing
two informations therefor, further, that they will be placed in double jeopardy.

ISSUE:
Whether or not the criminal prosecution of a case may be enjoined in this case.

RULING:
Yes. The Supreme Court rule in favor of Brocka, et al. and enjoin their criminal prosecution for the second
offense of inciting to sedition.

Indeed, the general rule is that criminal prosecution may not be restrained or stayed by injunction,
preliminary or final. There are however exceptions, among which are:

a. To afford adequate protection to the constitutional rights of the accused


b. When necessary for the orderly administration of justice or to avoid oppression or multiplicity of actions.
c. When there is a pre-judicial question which is sub judice
d. When the acts of the officer are without or in excess of authority
e. Where the prosecution is under an invalid law, ordinance or regulation
f. When double jeopardy is clearly apparent
g. Where the court has no jurisdiction over the offense h. Where it is a case of persecution rather than
prosecution
i. Where the charges are manifestly false and motivated by the lust for vengeance
j. When there is clearly no prima facie case against the accused and a Motion to quash on that ground
has been denied
k. Preliminary injunction has been issued by the Supreme Court to prevent the threatened unlawful arrest
of petitioners.

In the case at bar, criminal proceedings had become a case of persecution, have been undertaken by
state officials in bad faith. We, therefore, rule that where there is manifest bad faith that accompanies the
filing of criminal charges, as in the instant case where Brocka, et al. were barred from enjoying provisional
release until such time that charges were filed, and where a sham preliminary investigation was hastily
conducted, charges that are filed as a result should lawfully be enjoined.

E. Petition for Certiorari

1. Ty vs. NBI

FACTS:
Petitioners are stockholders of Omni Gas Corporation ("Omni"). They are being suspected of engaging in
illegal trading of petroleum products and underfilling of branded LPG cylinders in violation of B.P. 33, as
amended by P.D. 1865. NBI Agents Marvin De Jemil and Edgardo Kawada conducted surveillance
operations on Omni. On 15 April 2004, the NBI Agents carried out a test-buy. Using eight branded LPG
cylinders from Shell, Petron and Total, they went to Omni for refilling. Omni refilled the cylinders. The NBI
agents paid more than P1500. LPG Inspector Noel Navio found that the LPG cylinders were without LPG
valve seals and one of the cylinders was actually underfilled.

On 28 April 2004, Agent De Jemil obtained a search warrant from Pasig RTC branch 167. The NBI seized
several items from Omni's premises. Subsequently, Agent De Jemil filed his Complaint-Affidavit before
the DOJ. The Assistant City Prosecutor of Pasig found probable cause for violation of BP 33. This was
later approved by Chief State Prosecutor Jovencito Zuno.

Petitioners appealed the decision to the Secretary of Justice, who later reversed the decision of the Office
of the Chief State Prosecutor. NBI Agent De Jemil moved for reconsideration. Denied. He thus filed a
petition for certiorari under Rule 65 with the Court of Appeals.
The Court of Appeals affirmed the decision of Secretary of Justice. It later reversed itself and reinstated
the Resolution of the Chief State Prosecutor.

ISSUE:
Whether or not the petition for certiorari with the Court of Appeals was proper even if Agent De Jemil did
not appeal to the Office of the President

RULING:
YES. The determination of probable cause by the public prosecutor, and, later on, by the Secretary of
Justice, is subject to judicial review where it is established that grave abuse of discretion tainted the
determination. The aggrieved party need not resort to the Office of the President before availing of judicial
remedies because the Secretary of Justice is an alter ego of the President who may opt to exercise or not
to exercise his or her power of review over the former’s determination in criminal investigation cases.
Also, under the doctrine of qualified political agency, the determination of probable cause by the
Secretary of Justice is presumably that of the Chief Executive unless disapproved or reprobated by the
latter.

F. Bail

Effect of posting bail

Section 26, Rule 114. Bail not a bar to objections on illegal arrest, lack of or irregular preliminary
investigation. — An application for or admission to bail shall not bar the accused from challenging the
validity of his arrest or the legality of the warrant issued therefor, or from assailing the regularity or
questioning the absence of a preliminary investigation of the charge against him, provided that he raises
them before entering his plea. The court shall resolve the matter as early as practicable but not later than
the start of the trial of the case.

Cases:

1. Go vs. CA

FACTS:
Petitioner, while traveling in the wrong direction on a one-way street, almost had a collision with another
vehicle. Petitioner thereafter got out of his car, shot the driver of the other vehicle, and drove off. An
eyewitness of the incident was able to take down petitioner’s plate number and reported the same to the
police, who subsequently ordered a manhunt for petitioner. 6 days after the shooting, petitioner presented
himself in the police station, accompanied by 2 lawyers, the police detained him. Subsequently a criminal
charge was brought against him.

Petitioner posted bail, the prosecutor filed the case to the lower court, setting and commencing trial
without preliminary investigation. Prosecutor reasons that the petitioner has waived his right to preliminary
investigation as bail has been posted and that such situation, that petitioner has been arrested without a
warrant lawfully, falls under Section 5, Rule 113 and Section 7, Rule 112 of The 1985 Rules of Criminal
Procedure which provides for the rules and procedure pertaining to situations of lawful warrantless
arrests. Petitioner in his petition for certiorari assails such procedure and actions undertaken and files for
a preliminary investigation.

ISSUE:
WON petitioner effectively waived his right to preliminary investigation when he posted bail.

RULING:
No. Petitioner and prosecutor err in relying on Umil v. Ramos, wherein the Court upheld the warrantless
arrest as valid effected 1 to 14 days from actual commission of the offenses, which however constituted
“continuing crimes,” i.e. subversion, membership in an outlawed organization, etc. There was no lawful
warrantless arrest under Section 5, Rule 113. This is because the arresting officers were not actually
there during the incident, thus they had no personal knowledge and their information regarding petitioner
were derived from other sources. Further, Section 7, Rule 112, does not apply.

Petitioner was not arrested at all, as when he walked in the police station, he neither expressed surrender
nor any statement that he was or was not guilty of any crime.

Thus, when a complaint was filed to the prosecutor, preliminary investigation should have been
scheduled to determine probable cause. Prosecutor made a substantive error; petitioner is entitled to
preliminary investigation, necessarily in a criminal charge where the same is required appear thereat.

2. Larranaga vs. CA

FACTS:
On October 1, 1997, petitioner Margarita G. Larranaga filed a petition for certiorari, prohibition
and mandamus with writs of preliminary prohibitory and mandatory injunction seeking to annul the
information for kidnapping and serious illegal detention against her minor son, Francisco Juan
Larranagga alias Paco, filed in the RTC of Cebu City as well as the warrant of arrest issued as a
consequence thereof. On October 6, 1997, petitioner filed a Supplemental Petition asking for the
issuance of the writ of habeas corpus to relieve her son from his alleged illegal confinement or to grant
him bail.

The prosecutor denied this request, arguing that Larranaga was entitled only to an inquest investigation.
On September 19, 1997, Larranaga’s counsel appealed to the Court of Appeals to prevent the filing of
criminal information against Larranaga. However, criminal charges had already been filed on September
17, 1997 with the Regional Trial Court of Cebu City. On September 22, 1997, counsel filed a petition with
the Court of Appeals requesting that the Regional Trial Court of Cebu City prevent Larranaga’s arrest.
Nevertheless, he was arrested on that day with a warrant issued by the Executive Judge of the RTC of
Cebu City, the Honorable Priscilla Agana.

Another petition was filed in the Court of Appeals against his arrest and dismissed on September 25,
1997. This decision was appealed to the Supreme Court. Despite this pending appeal, Larranaga was
brought before a judge on October 14, 1997. He did not enter a plea and the judge thus entered a plea of
not guilty to two counts of kidnapping with serious illegal detention.
On October 16, 1997, the Supreme Court temporarily restrained this judge from proceeding with the case
to prevent the issues before the court from becoming moot.

ISSUE:
(1)Whether petitioner must be released in the absence of information and warrant of arrest.
(2)Whether petitioner may be allowed to post bail

RULING:
(1)No. The filing of charges and the issuance of the warrant of arrest against a person invalidly detained
will cure the defect of that detention or at least deny him the right to be released because of such defect.
The original warrantless arrest of the petitioner was doubtless illegal. Nevertheless, the
Regional Trial Court lawfully acquired jurisdiction over the person of the petitioner by virtue of the warrant
of arrest it issued on August 26, 1993 against him and the other accused in connection with the rape-slay
cases. It was belated, to be sure, but it was nonetheless legal.

(2) As to the issue of bail, the State party explains that no bail shall be granted where an accused is
charged with an offence punishable by death or life imprisonment, and the evidence of guilt is strong.

Das könnte Ihnen auch gefallen